Location via proxy:   [ UP ]  
[Report a bug]   [Manage cookies]                

Series 3

Download as pdf or txt
Download as pdf or txt
You are on page 1of 219

1. Which of the following is not a feature of mumps?

a. Caused by paramyxovirus
b. Aseptic meningitis is common in children
c. Orchitis is a complication in adults
d. Incubational period is less than 2 weeks
Answer: d. Incubational period is less than 2 weeks

2. Constipation may occur as a side effect of:

a. Carbenoxolone
b. Ranitidine
c. De-glycerinised liquors
d. Sucrose octa-sulphate
Answer: a. Carbenoxolone

3. Rheumatic nodules:

a. Are painful
b. Are attached to skin
c. Are more common in adults than in children
d. Rarely occur unless rheumatic fever is present
Answer: d. Rarely occur unless rheumatic fever is present

4. The antibiotic which is safe in renal failure is:

a. Kanamycin
b. Penicillin
c. Minocycline
d. Gentamicin
Answer: b. Penicillin

5. Sudden death is common in:

a. Mitral stenosis
b. Aortic stenosis
c. VSD
d. PDA
Answer: b. Aortic stenosis

6. Which of the following does not cause tachycardia?

a. Obstructive jaundice
b. Shock
c. Thyrotoxicosis
d. Digitalis poisoning
Answer: d. Digitalis poisoning

7. Which of the following is a common side effect of long-term corticosteroid


therapy?

a. Hypocalcemia
b. Weight gain
c. Hyperkalemia
d. Hypoglycemia
Answer: b. Weight gain

8. The first-line treatment for hypertension in pregnancy is:

a. Methyldopa
b. Labetalol
c. Amlodipine
d. Enalapril
Answer: a. Labetalol

9. The most common cause of acute pancreatitis is:

a. Alcohol consumption
b. Gallstones
c. Hyperlipidemia
d. Trauma
Answer: b. Gallstones

10. Which of the following is a characteristic of Hodgkin’s lymphoma?


a. Age of onset is usually in older adults
b. Presence of Reed-Sternberg cells
c. More common in males than females
d. Linked to HIV infection
Answer: b. Presence of Reed-Sternberg cells

11. A patient presents with a triad of symptoms: fever, jaundice, and right upper
quadrant pain. This is suggestive of:

a. Cholecystitis
b. Acute hepatitis
c. Ascending cholangitis
d. Pancreatitis
Answer: c. Ascending cholangitis

12. In the case of anaphylaxis, the first-line treatment is:

a. Antihistamines
b. Corticosteroids
c. Epinephrine
d. Saline infusion
Answer: c. Epinephrine

13. Which of the following is a common complication of diabetes mellitus?

a. Hyperthyroidism
b. Retinopathy
c. Osteoporosis
d. Psoriasis
Answer: b. Retinopathy

14. A 70-year-old male presents with sudden onset of chest pain and
diaphoresis. The most likely diagnosis is:

a. Angina pectoris
b. Myocardial infarction
c. Aortic dissection
d. Pulmonary embolism
Answer: b. Myocardial infarction

15. The most common type of lung cancer is:

a. Small cell carcinoma


b. Squamous cell carcinoma
c. Adenocarcinoma
d. Large cell carcinoma
Answer: c. Adenocarcinoma

16. In patients with rheumatoid arthritis, the characteristic laboratory finding


is:

a. Elevated blood glucose


b. Positive rheumatoid factor
c. Low hemoglobin
d. Elevated creatinine
Answer: b. Positive rheumatoid factor

17. The primary function of the kidney is:

a. Filtration of toxins
b. Regulation of blood pressure
c. Regulation of electrolytes
d. All of the above
Answer: d. All of the above

18. Which of the following drugs is used to treat acute migraine attacks?

a. Amitriptyline
b. Propranolol
c. Sumatriptan
d. Topiramate
Answer: c. Sumatriptan

19. A child with a history of recurrent infections may have:


a. Hypertension
b. Immunodeficiency
c. Hyperthyroidism
d. Diabetes insipidus
Answer: b. Immunodeficiency

20. The main organ affected in Wilson's disease is:

a. Liver
b. Heart
c. Brain
d. Kidneys
Answer: a. Liver

21. The most common site of ectopic pregnancy is:

a. Ovaries
b. Cervix
c. Fallopian tubes
d. Abdominal cavity
Answer: c. Fallopian tubes

22. A common side effect of warfarin therapy is:

a. Thrombosis
b. Hemorrhage
c. Hyperlipidemia
d. Hypertension
Answer: b. Hemorrhage

23. Which of the following is not a common symptom of congestive heart


failure?

a. Dyspnea
b. Edema
c. Chest pain
d. Fatigue
Answer: c. Chest pain

24. The most sensitive test for detecting myocardial infarction is:

a. ECG
b. Troponin levels
c. CK-MB
d. Myoglobin
Answer: b. Troponin levels

25. The classic triad of symptoms in pheochromocytoma includes:

a. Hypertension, palpitations, sweating


b. Abdominal pain, vomiting, diarrhea
c. Fever, rash, joint pain
d. Weight loss, hyperglycemia, polyuria
Answer: a. Hypertension, palpitations, sweating

26. The primary cause of peptic ulcers is:

a. Stress
b. H. pylori infection
c. Alcohol consumption
d. Smoking
Answer: b. H. pylori infection

27. In acute appendicitis, the most common initial symptom is:

a. Diarrhea
b. Nausea
c. Abdominal pain
d. Fever
Answer: c. Abdominal pain

28. The most common inherited bleeding disorder is:


a. Hemophilia A
b. von Willebrand disease
c. Thrombocytopenia
d. Disseminated intravascular coagulation
Answer: b. von Willebrand disease

29. A significant risk factor for developing type 2 diabetes mellitus is:

a. Age under 30
b. Sedentary lifestyle
c. High fiber diet
d. Regular exercise
Answer: b. Sedentary lifestyle

30. Which of the following is not a typical feature of Parkinson's disease?

a. Bradykinesia
b. Rigidity
c. Tachycardia
d. Resting tremor
Answer: c. Tachycardia

31. Which of the following conditions is characterized by "cherry-red spot" on


the macula?

a. Retinitis pigmentosa
b. Tay-Sachs disease
c. Diabetic retinopathy
d. Glaucoma
Answer: b. Tay-Sachs disease

32. A common presenting symptom of hyperthyroidism is:

a. Weight gain
b. Cold intolerance
c. Heat intolerance
d. Bradycardia
Answer: c. Heat intolerance

33. The primary cause of secondary hypertension is:

a. Obesity
b. Renal disease
c. Stress
d. Sedentary lifestyle
Answer: b. Renal disease

34. In patients with chronic obstructive pulmonary disease (COPD), the most
common symptom is:

a. Cough
b. Chest pain
c. Wheezing
d. Hemoptysis
Answer: a. Cough

35. Which of the following vaccines is contraindicated in pregnancy?

a. Influenza vaccine
b. Tdap vaccine
c. MMR vaccine
d. Hepatitis B vaccine
Answer: c. MMR vaccine

36. The classic symptom of multiple sclerosis is:

a. Visual disturbances
b. Muscle weakness
c. Tremors
d. Gait instability
Answer: a. Visual disturbances

37. The most appropriate initial management for anaphylactic shock is:
a. Intravenous fluids
b. Epinephrine
c. Antihistamines
d. Corticosteroids
Answer: b. Epinephrine

38. A patient with renal failure is likely to have which electrolyte imbalance?

a. Hypercalcemia
b. Hypernatremia
c. Hyperkalemia
d. Hypermagnesemia
Answer: c. Hyperkalemia

39. The hallmark symptom of diabetic ketoacidosis (DKA) is:

a. Polyuria
b. Acidosis
c. Kussmaul respirations
d. Dehydration
Answer: c. Kussmaul respirations

40.Which of the following is a common cause of hemolytic anemia?

a. Iron deficiency
b. Sickle cell disease
c. Thalassemia
d. Vitamin B12 deficiency
Answer: b. Sickle cell disease

41. A common finding in a patient with Cushing's syndrome is:

a. Hypotension
b. Moon facies
c. Hyperkalemia
d. Weight loss
Answer: b. Moon facies
42. The most common cause of secondary amenorrhea is:

a. Pregnancy
b. Ovarian cysts
c. Stress
d. Thyroid disorders
Answer: a. Pregnancy

43. The primary action of insulin is to:

a. Increase blood glucose levels


b. Decrease blood glucose levels
c. Stimulate glucagon release
d. Promote ketogenesis
Answer: b. Decrease blood glucose levels

44.Which condition is associated with a "steeple sign" on an X-ray?

a. Croup
b. Epiglottitis
c. Asthma
d. Pneumonia
Answer: a. Croup

45. The best initial test to diagnose deep vein thrombosis (DVT) is:

a. Ultrasound
b. MRI
c. CT scan
d. D-dimer
Answer: a. Ultrasound

46.The classic triad of symptoms for meningitis includes:

a. Fever, headache, neck stiffness


b. Cough, fever, wheezing
c. Jaundice, fever, abdominal pain
d. Pain, pallor, pulselessness
Answer: a. Fever, headache, neck stiffness

47. In primary adrenal insufficiency, which hormone is deficient?

a. Cortisol
b. Aldosterone
c. Androgens
d. All of the above
Answer: d. All of the above

48.The most common cause of peptic ulcer disease is:

a. NSAIDs
b. H. pylori infection
c. Stress
d. Alcohol
Answer: b. H. pylori infection

49.A classic feature of diabetic neuropathy is:

a. Sensory loss in a stocking-glove distribution


b. Muscle weakness
c. Flaccid paralysis
d. Reflex hyperactivity
Answer: a. Sensory loss in a stocking-glove distribution

50. The primary goal of treatment in congestive heart failure is to:

a. Increase blood pressure


b. Decrease fluid retention
c. Increase heart rate
d. Lower blood glucose
Answer: b. Decrease fluid retention

51. The most common site of atherosclerosis is:


a. Aorta
b. Coronary arteries
c. Carotid arteries
d. Peripheral arteries
Answer: b. Coronary arteries

52. In chronic renal failure, which of the following is commonly seen?

a. Hypercalcemia
b. Metabolic acidosis
c. Hypernatremia
d. Hypokalemia
Answer: b. Metabolic acidosis

53. The first-line treatment for hypertension in a diabetic patient is:

a. Calcium channel blockers


b. ACE inhibitors
c. Beta-blockers
d. Diuretics
Answer: b. ACE inhibitors

54. A common side effect of benzodiazepines is:

a. Hypertension
b. Drowsiness
c. Bradycardia
d. Hyperactivity
Answer: b. Drowsiness

55. The typical presentation of a patient with asthma includes:

a. Persistent cough
b. Chest tightness
c. Wheezing
d. All of the above
Answer: d. All of the above
56. The most appropriate initial treatment for a patient with acute coronary
syndrome is:

a..Morphine
b. Oxygen
c. Aspirin
d. Nitroglycerin
Answer: c. Aspirin

57. A common feature of chronic liver disease is:

a. Hyperglycemia
b. Ascites
c. Hypokalemia
d. Bradycardia
Answer: b. Ascites

58. In cases of suspected acute appendicitis, the most reliable physical exam
finding is:

a. Rebound tenderness
b. Murphy’s sign
c. Rovsing’s sign
d. Psoas sign
Answer: a. Rebound tenderness

59. Which of the following is a common cause of secondary hypertension?

a. Obstructive sleep apnea


b. Hyperlipidemia
c. Smoking
d. Alcohol consumption
Answer: a. Obstructive sleep apnea

60.The most common cause of hemoptysis is:


a. Pulmonary embolism
b. Tuberculosis
c. Lung cancer
d. Bronchitis
Answer: b. Tuberculosis

61. A patient presents with fatigue, pallor, and a "beefy" red tongue. This is
indicative of:

a. Iron deficiency anemia


b. Folic acid deficiency
c. Vitamin B12 deficiency
d. Sickle cell anemia
Answer: c. Vitamin B12 deficiency

62. The best initial treatment for a patient with an acute asthma attack is:

a. Inhaled corticosteroids
b. Long-acting beta agonists
c. Short-acting beta agonists
d. Anticholinergics
Answer: c. Short-acting beta agonists

63. The classic triad of symptoms in osteoarthritis includes:

a. Pain, swelling, and redness


b. Pain, stiffness, and functional impairment
c. Pain, deformity, and fatigue
d. Pain, fever, and malaise
Answer: b. Pain, stiffness, and functional impairment

64.The first-line treatment for major depressive disorder is:

a. Benzodiazepines
b. Antidepressants
c. Antipsychotics
d. Mood stabilizers
Answer: b. Antidepressants

65. A patient with severe hypertension and headache is likely suffering from:

a. Hypertensive crisis
b. Stroke
c. Heart failure
d. Myocardial infarction
Answer: a. Hypertensive crisis

66.In a patient with congestive heart failure, the most appropriate medication
to reduce preload is:

a. Digoxin
b. Loop diuretics
c. ACE inhibitors
d. Beta-blockers
Answer: b. Loop diuretics

67. Which of the following is a characteristic sign of nephrotic syndrome?

a. Hematuria
b. Proteinuria
c. Hypertension
d. Oliguria
Answer: b. Proteinuria

68.A common cause of pneumonia in children is:

a. Staphylococcus aureus
b. Streptococcus pneumoniae
c. Mycoplasma pneumoniae
d. Klebsiella pneumoniae
Answer: b. Streptococcus pneumoniae

69.The primary symptom of aortic stenosis is:


a. Chest pain
b. Syncope
c. Dyspnea on exertion
d. Palpitations
Answer: c. Dyspnea on exertion

70. The most common type of thyroid carcinoma is:

a. Papillary carcinoma
b. Follicular carcinoma
c. Medullary carcinoma
d. Anaplastic carcinoma
Answer: a. Papillary carcinoma

71. In which condition would you expect a "sausage-shaped" abdomen in


children? a. Intussusception
b. Appendicitis
c. Volvulus
d. Hernia
Answer: a. Intussusception
72. A patient with "moon facies" and "buffalo hump" is likely suffering from:

a. Cushing's syndrome
b. Addison's disease
c. Acromegaly
d. Hyperthyroidism
Answer: a. Cushing's syndrome

73. The hallmark sign of hyperparathyroidism is:

a. Hypocalcemia
b. Hypercalcemia
c. Hyperkalemia
d. Hyponatremia
Answer: b. Hypercalcemia
74. The most common type of stroke is:

a. Ischemic stroke
b. Hemorrhagic stroke
c. Transient ischemic attack (TIA)
d. Cryptogenic stroke
Answer: a. Ischemic stroke

75. Which of the following conditions is associated with a high risk of


developing gastric cancer?

a. Chronic gastritis
b. Peptic ulcer disease
c. Gastroesophageal reflux disease (GERD)
d. Gallbladder disease
Answer: a. Chronic gastritis

76. The primary treatment for a patient diagnosed with a tension pneumothorax
is:

a. Thoracotomy
b. Chest tube insertion
c. Needle decompression
d. Antibiotics
Answer: c. Needle decompression

77. The drug of choice for status epilepticus is:

a. Carbamazepine
b. Phenytoin
c. Valproate
d. Lorazepam
Answer: d. Lorazepam

78. The primary characteristic of Crohn's disease is:


a. Continuous lesions
b. Involvement of the rectum
c. Skip lesions
d. Superficial inflammation
Answer: c. Skip lesions

79. The most common cause of acute gastroenteritis in children is:

a. Rotavirus
b. Norovirus
c. Salmonella
d. E. coli
Answer: a. Rotavirus

80.A classic sign of Addison's disease is:

a. Hyperpigmentation
b. Hypotension
c. Hyponatremia
d. All of the above
Answer: d. All of the above

81. The most common cause of acute pancreatitis is:

a. Alcohol consumption
b. Gallstones
c. Hyperlipidemia
d. Trauma
Answer: b. Gallstones

82. In diabetes mellitus, the primary cause of neuropathy is:

a. Vascular damage
b. Direct nerve injury
c. Hypoxia
d. Inflammation
Answer: a. Vascular damage
83. Which vitamin deficiency is associated with night blindness?

a. Vitamin A
b. Vitamin B12
c. Vitamin C
d. Vitamin D
Answer: a. Vitamin A

84.The classic triad of symptoms in pheochromocytoma includes:

a. Headache, palpitations, sweating


b. Abdominal pain, diarrhea, fever
c. Weight loss, fatigue, hyperpigmentation
d. Polyuria, polydipsia, weight gain
Answer: a. Headache, palpitations, sweating

85. Which of the following is a risk factor for developing deep vein thrombosis
(DVT)?

a. Smoking
b. Sedentary lifestyle
c. Recent surgery
d. All of the above
Answer: d. All of the above

86.A child presents with a "barking" cough and stridor. This is indicative of:

a. Croup
b. Epiglottitis
c. Bronchiolitis
d. Asthma
Answer: a. Croup

87. Which of the following is a common finding in rheumatoid arthritis?

a. Morning stiffness
b. Symmetrical joint involvement
c. Rheumatoid nodules
d. All of the above
Answer: d. All of the above

88. The first-line treatment for hypertension in the elderly is:

a. Beta-blockers
b. ACE inhibitors
c. Diuretics
d. Calcium channel blockers
Answer: c. Diuretics

89.A characteristic feature of viral hepatitis is:

a. Increased alkaline phosphatase


b. Increased bilirubin
c. Decreased AST and ALT
d. Decreased liver size
Answer: b. Increased bilirubin

90.The drug of choice for the treatment of tuberculosis is:

a. Rifampin
b. Isoniazid
c. Pyrazinamide
d. Ethambutol
Answer: a. Rifampin

91. A common sign of liver cirrhosis is:

a. Ascites
b. Hypertension
c. Jaundice
d. All of the above
Answer: d. All of the above

92. The most common infectious cause of myocarditis is:


a. Streptococcus
b. Staphylococcus
c. Coxsackie virus
d. Influenza virus
Answer: c. Coxsackie virus

93. The characteristic sign of Guillain-Barré syndrome is:

a. Muscle atrophy
b. Ascending paralysis
c. Flaccid paralysis
d. Tremors
Answer: b. Ascending paralysis

94.The most appropriate first-line treatment for hyperlipidemia is:

a. Statins
b. Fibrates
c. Niacin
d. Bile acid sequestrants
Answer: a. Statins

95. The most common cause of secondary amenorrhea is:

a. Pregnancy
b. Ovarian failure
c. Thyroid disorders
d. Anorexia nervosa
Answer: a. Pregnancy

96.Which of the following is a common symptom of hyperthyroidism?

a. Weight gain
b. Fatigue
c. Heat intolerance
d. Depression
Answer: c. Heat intolerance
97. The classic symptom of meningococcal meningitis is:

a. Neck stiffness
b. Rash
c. Fever
d. All of the above
Answer: d. All of the above

98.The most common type of skin cancer is:

a. Melanoma
b. Basal cell carcinoma
c. Squamous cell carcinoma
d. Merkel cell carcinoma
Answer: b. Basal cell carcinoma

99.The primary complication of untreated hypertension is:

a. Stroke
b. Heart failure
c. Renal failure
d. All of the above
Answer: d. All of the above

100. The best initial test for diagnosing diabetes mellitus is:

a. Fasting plasma glucose


b. Oral glucose tolerance test
c. Hemoglobin A1c
d. Random plasma glucose
Answer: a. Fasting plasma glucose

101. Which of the following is a common feature of schizophrenia?

a. Hallucinations
b. Delusions
c. Disorganized thinking
d. All of the above
Answer: d. All of the above

102. The most common symptom of cervical cancer is:

a. Abnormal vaginal bleeding


b. Pelvic pain
c. Weight loss
d. Dysuria
Answer: a. Abnormal vaginal bleeding

103. A patient presents with fever, jaundice, and abdominal pain. The most
likely diagnosis is:

a. Hepatitis
b. Cholecystitis
c. Pancreatitis
d. Cirrhosis
Answer: a. Hepatitis

104. The first-line treatment for allergic rhinitis is:

a. Antihistamines
b. Decongestants
c. Corticosteroids
d. Leukotriene receptor antagonists
Answer: c. Corticosteroids

105. Which of the following is a complication of diabetes mellitus?

a. Diabetic retinopathy
b. Diabetic neuropathy
c. Diabetic nephropathy
d. All of the above
Answer: d. All of the above

106. The characteristic feature of osteoarthritis is:


a. Symmetrical joint involvement
b. Heberden's nodes
c. Morning stiffness lasting more than an hour
d. Erythema and swelling
Answer: b. Heberden's nodes

107. The most common pathogen in community-acquired pneumonia is:

a. Streptococcus pneumoniae
b. Haemophilus influenzae
c. Mycoplasma pneumoniae
d. Chlamydia pneumoniae
Answer: a. Streptococcus pneumoniae

108. A patient with a history of excessive alcohol use presents with confusion
and ataxia. This is indicative of:

a. Wernicke's encephalopathy
b. Korsakoff syndrome
c. Hepatic encephalopathy
d. Delirium tremens
Answer: a. Wernicke's encephalopathy

109. The most common initial symptom of Parkinson's disease is:

a. Tremor
b. Rigidity
c. Bradykinesia
d. Postural instability
Answer: a. Tremor

110. The most appropriate initial test for suspected chronic obstructive
pulmonary disease (COPD) is:

a. Chest X-ray
b. CT scan
c. Spirometry
d. Arterial blood gas analysis
Answer: c. Spirometry

111.The characteristic feature of systemic lupus erythematosus (SLE) is:

a. Malar rash
b. Discoid rash
c. Photosensitivity
d. All of the above
Answer: d. All of the above

112. A patient with sudden onset of severe chest pain, dyspnea, and
tachycardia is likely suffering from:

a. Pulmonary embolism
b. Myocardial infarction
c. Aortic dissection
d. Pneumothorax
Answer: a. Pulmonary embolism

113. The primary action of statins is to:

a. Lower triglycerides
b. Lower LDL cholesterol
c. Raise HDL cholesterol
d. Prevent platelet aggregation
Answer: b. Lower LDL cholesterol

114. The most common cause of peptic ulcers is:

a. Stress
b. H. pylori infection
c. NSAID use
d. Alcohol consumption
Answer: b. H. pylori infection

115. The most common cause of chronic cough in adults is:


a. Asthma
b. COPD
c. GERD
d. Post-nasal drip
Answer: d. Post-nasal drip

116. The most common cause of acute kidney injury is:

a. Pre-renal causes
b. Intrinsic renal causes
c. Post-renal causes
d. Obstruction
Answer: a. Pre-renal causes

117. The primary mechanism of action of ACE inhibitors is:

a. Vasodilation
b. Decreased heart rate
c. Decreased renal blood flow
d. Increased sodium retention
Answer: a. Vasodilation

118. A common side effect of long-term corticosteroid use is:

a. Osteoporosis
b. Hypertension
c. Hyperglycemia
d. All of the above
Answer: d. All of the above

119. The classic symptom of appendicitis is:

a. Right upper quadrant pain


b. Left lower quadrant pain
c. Periumbilical pain
d. Flank pain
Answer: c. Periumbilical pain
120. Which of the following is associated with an increased risk of developing
breast cancer?

a. Early menarche
b. Late menopause
c. Family history
d. All of the above
Answer: d. All of the above

121. The most common cause of hypercalcemia is:

a. Primary hyperparathyroidism
b. Malignancy
c. Vitamin D intoxication
d. Sarcoidosis
Answer: a. Primary hyperparathyroidism

122. A classic sign of nephrotic syndrome is:

a. Hematuria
b. Hypertension
c. Proteinuria
d. Oliguria
Answer: c. Proteinuria

123. The primary treatment for anaphylaxis is:

a. Antihistamines
b. Corticosteroids
c. Epinephrine
d. Saline
Answer: c. Epinephrine

124. The most common cause of chronic liver disease worldwide is:

a. Alcohol use
b. Non-alcoholic fatty liver disease
c. Hepatitis B
d. Hepatitis C
Answer: b. Non-alcoholic fatty liver disease

125. Which of the following is a common finding in hypothyroidism?

a. Weight loss
b. Tachycardia
c. Cold intolerance
d. Increased appetite
Answer: c. Cold intolerance

126. The drug of choice for hypertension in pregnancy is:

a. Labetalol
b. Enalapril
c. Amlodipine
d. Hydrochlorothiazide
Answer: a. Labetalol

127. The most common type of stroke is:

a. Hemorrhagic stroke
b. Ischemic stroke
c. Transient ischemic attack
d. Subarachnoid hemorrhage
Answer: b. Ischemic stroke

128. A patient with a sudden onset of severe headache and neck stiffness
likely has:

a. Migraine
b. Tension headache
c. Meningitis
d. Cluster headache
Answer: c. Meningitis
129. Which of the following is a typical feature of Alzheimer's disease?

a. Memory loss
b. Disorientation
c. Personality changes
d. All of the above
Answer: d. All of the above

130. The primary mechanism of action of metformin is:

a. Increased insulin secretion


b. Decreased hepatic glucose production
c. Increased glucose absorption in the intestines
d. Decreased insulin resistance
Answer: b. Decreased hepatic glucose production

131. The classic triad of symptoms in aortic stenosis includes:

a. Chest pain, dyspnea, syncope


b. Palpitations, fatigue, edema
c. Cough, wheezing, fever
d. Headache, nausea, vomiting
Answer: a. Chest pain, dyspnea, syncope

132. Which of the following is a classic symptom of asthma?

a. Chest tightness
b. Weight loss
c. Fever
d. Hemoptysis
Answer: a. Chest tightness

133. The most effective long-term treatment for asthma is:

a. Short-acting beta-agonists
b. Inhaled corticosteroids
c. Leukotriene modifiers
d. Antihistamines
Answer: b. Inhaled corticosteroids

134. A common side effect of long-term use of proton pump inhibitors is:

a. Osteoporosis
b. Renal failure
c. Hypertension
d. Hyperkalemia
Answer: a. Osteoporosis

135. Which of the following is a sign of acute cholecystitis?

a. Murphy's sign
b. McBurney's point tenderness
c. Positive Babinski reflex
d. Costovertebral angle tenderness
Answer: a. Murphy's sign

136. The most common cause of secondary hypertension is:

a. Kidney disease
b. Endocrine disorders
c. Obstructive sleep apnea
d. All of the above
Answer: d. All of the above

137. Which of the following conditions is associated with increased risk of


thromboembolism?

a. Varicose veins
b. Atrial fibrillation
c. Immobility
d. All of the above
Answer: d. All of the above

138. The first-line treatment for chronic gout is:


a. Allopurinol
b. Colchicine
c. Indomethacin
d. Probenecid
Answer: a. Allopurinol

139. Which of the following is a classic symptom of chronic obstructive


pulmonary disease (COPD)?

a. Cough
b. Dyspnea
c. Wheezing
d. All of the above
Answer: d. All of the above

140. The most common pathogen causing urinary tract infections in women
is:

a. Staphylococcus saprophyticus
b. Escherichia coli
c. Klebsiella pneumoniae
d. Proteus mirabilis
Answer: b. Escherichia coli

141. The characteristic symptom of celiac disease is:

a. Abdominal pain
b. Diarrhea
c. Weight loss
d. All of the above
Answer: d. All of the above

142. Which of the following is commonly associated with Marfan syndrome?


a. Short stature
b. Aortic dilation
c. Hyperpigmentation
d. Osteoporosis
Answer: b. Aortic dilation
143. The most common cause of a sudden, severe headache is:

a. Migraine
b. Tension headache
c. Subarachnoid hemorrhage
d. Cluster headache
Answer: c. Subarachnoid hemorrhage

144. The most appropriate screening test for breast cancer is:

a. Mammography
b. Ultrasound
c. MRI
d. Clinical breast examination
Answer: a. Mammography

145. The most common presenting symptom of colorectal cancer is:

a. Weight loss
b. Abdominal pain
c. Change in bowel habits
d. Rectal bleeding
Answer: d. Rectal bleeding

146. A patient presents with fatigue, pallor, and elevated liver enzymes. The
most likely diagnosis is:

a. Hemolytic anemia
b. Chronic liver disease
c. Iron deficiency anemia
d. Sickle cell disease
Answer: b. Chronic liver disease

147. The most common cause of hypothyroidism is:


a. Hashimoto's thyroiditis
b. Iodine deficiency
c. Thyroidectomy
d. Radiation therapy
Answer: a. Hashimoto's thyroiditis

148. The classic triad of symptoms in congenital adrenal hyperplasia includes:

a. Hypertension, hypokalemia, hirsutism


b. Ambiguous genitalia, adrenal crisis, hirsutism
c. Weight gain, fatigue, cold intolerance
d. Cough, hemoptysis, fever
Answer: b. Ambiguous genitalia, adrenal crisis, hirsutism

149. Which of the following is a common side effect of ACE inhibitors?

a. Cough
b. Hyperkalemia
c. Angioedema
d. All of the above
Answer: d. All of the above

150. The primary mechanism of action of aspirin in cardiovascular disease is:

a. Platelet aggregation inhibition


b. Vasodilation
c. Decreased heart rate
d. Increased HDL cholesterol
Answer: a. Platelet aggregation inhibition

151. The most common type of lung cancer is:

a. Squamous cell carcinoma


b. Adenocarcinoma
c. Small cell carcinoma
d. Large cell carcinoma
Answer: b. Adenocarcinoma
152. A characteristic feature of diabetic ketoacidosis (DKA) is:

a. Hyperglycemia
b. Acidosis
c. Ketosis
d. All of the above
Answer: d. All of the above

153. The primary site of absorption for most nutrients in the gastrointestinal
tract is:

a. Stomach
b. Duodenum
c. Jejunum
d. Ileum
Answer: c. Jejunum

154. The leading cause of chronic liver disease in the United States is:

a. Alcohol abuse
b. Hepatitis C
c. Non-alcoholic fatty liver disease
d. Hepatitis B
Answer: c. Non-alcoholic fatty liver disease

155. A common side effect of beta-blockers is:

a. Bradycardia
b. Tachycardia
c. Hyperkalemia
d. Hypertension
Answer: a. Bradycardia

156. The drug of choice for hypertension in a patient with heart failure is:

a. Calcium channel blockers


b. Diuretics
c. ACE inhibitors
d. Beta-blockers
Answer: c. ACE inhibitors

157. The hallmark symptom of chronic pancreatitis is:

a. Epigastric pain
b. Jaundice
c. Weight gain
d. Steatorrhea
Answer: a. Epigastric pain

158. The most common infectious agent in osteomyelitis is:

a. Staphylococcus aureus
b. Streptococcus pneumoniae
c. Escherichia coli
d. Pseudomonas aeruginosa
Answer: a. Staphylococcus aureus

159. The main side effect of clindamycin is:

a. Nephrotoxicity
b. Gastrointestinal disturbances
c. Neurotoxicity
d. Hematological toxicity
Answer: b. Gastrointestinal disturbances

160. A positive Chvostek sign indicates:

a. Hyperkalemia
b. Hypercalcemia
c. Hypoparathyroidism
d. Hypocalcemia
Answer: d. Hypocalcemia

161. The most common type of arrhythmia in patients with heart failure is:
a. Atrial fibrillation
b. Ventricular tachycardia
c. Premature ventricular contractions
d. Supraventricular tachycardia
Answer: a. Atrial fibrillation

162. Which of the following is the primary cause of primary adrenal


insufficiency (Addison's disease)?

a. Autoimmune destruction
b. Infectious disease
c. Tumors
d. Genetic disorders
Answer: a. Autoimmune destruction

163. The classic symptom of multiple myeloma is:

a. Hypercalcemia
b. Anemia
c. Bone pain
d. All of the above
Answer: d. All of the above

164. The most common cause of hypopituitarism is:

a. Pituitary adenoma
b. Sheehan's syndrome
c. Craniopharyngioma
d. Sarcoidosis
Answer: a. Pituitary adenoma

165. The drug of choice for generalized anxiety disorder is:

a. Fluoxetine
b. Sertraline
c. Buspirone
d. Diazepam
Answer: c. Buspirone

166. The most common pathogen causing community-acquired pneumonia


is:

a. Streptococcus pneumoniae
b. Haemophilus influenzae
c. Mycoplasma pneumoniae
d. Chlamydia pneumoniae
Answer: a. Streptococcus pneumoniae

167. Which of the following is the primary treatment for acute sinusitis?

a. Antibiotics
b. Nasal decongestants
c. Surgery
d. Antihistamines
Answer: a. Antibiotics

168. The primary risk factor for developing deep vein thrombosis is:

a. Age
b. Immobility
c. Obesity
d. All of the above
Answer: d. All of the above

169. The first-line treatment for major depressive disorder is:

a. Psychotherapy
b. Electroconvulsive therapy
c. Selective serotonin reuptake inhibitors (SSRIs)
d. Tricyclic antidepressants
Answer: c. Selective serotonin reuptake inhibitors (SSRIs)

170. Which of the following is a common complication of diabetes mellitus?


a. Neuropathy
b. Nephropathy
c. Retinopathy
d. All of the above
Answer: d. All of the above

171. The most common type of heart failure is:

a. Systolic heart failure


b. Diastolic heart failure
c. Right-sided heart failure
d. Left-sided heart failure
Answer: a. Systolic heart failure

172. Which of the following tests is most useful for diagnosing pneumonia?

a. Chest X-ray
b. CT scan
c. Sputum culture
d. Bronchoscopy
Answer: a. Chest X-ray

173. The primary treatment for rheumatoid arthritis includes:

a. Non-steroidal anti-inflammatory drugs (NSAIDs)


b. Disease-modifying antirheumatic drugs (DMARDs)
c. Corticosteroids
d. All of the above
Answer: d. All of the above

174. The characteristic "barking" cough in children is associated with:

a. Bronchiolitis
b. Asthma
c. Croup
d. Pneumonia
Answer: c. Croup
175. The main mechanism of action of statins is:

a. Decrease triglycerides
b. Increase HDL cholesterol
c. Inhibit HMG-CoA reductase
d. Inhibit absorption of cholesterol
Answer: c. Inhibit HMG-CoA reductase

176. The classic triad of symptoms for pheochromocytoma includes:

a. Headache, palpitations, sweating


b. Weight loss, fatigue, hyperglycemia
c. Hypertension, tachycardia, fever
d. Abdominal pain, diarrhea, flushing
Answer: a. Headache, palpitations, sweating

177. Which of the following is a common complication of hypertension?

a. Stroke
b. Heart failure
c. Chronic kidney disease
d. All of the above
Answer: d. All of the above

178. A characteristic feature of hyperthyroidism is:

a. Weight gain
b. Cold intolerance
c. Tachycardia
d. Dry skin
Answer: c. Tachycardia

179. The first-line treatment for acute migraine attacks is:

a. NSAIDs
b. Triptans
c. Opioids
d. Antiemetics
Answer: b. Triptans

180. Which of the following is a classic finding in chronic kidney disease?

a. Anemia
b. Hyperkalemia
c. Edema
d. All of the above
Answer: d. All of the above

181. The most common type of thyroid cancer is:

a. Papillary carcinoma
b. Follicular carcinoma
c. Medullary carcinoma
d. Anaplastic carcinoma
Answer: a. Papillary carcinoma

182. Which of the following medications is contraindicated in pregnancy?

a. Acetaminophen
b. Metformin
c. Warfarin
d. Insulin
Answer: c. Warfarin

183. The primary mechanism of action of ACE inhibitors is:

a. Blocking calcium channels


b. Inhibiting the conversion of angiotensin I to angiotensin II
c. Blocking beta-adrenergic receptors
d. Increasing renal blood flow
Answer: b. Inhibiting the conversion of angiotensin I to angiotensin II

184. The most common form of anemia in the general population is:
a. Iron deficiency anemia
b. Vitamin B12 deficiency anemia
c. Aplastic anemia
d. Hemolytic anemia
Answer: a. Iron deficiency anemia

185. A classic sign of meningitis is:

a. Brudzinski's sign
b. Murphy's sign
c. McBurney's point tenderness
d. Homan's sign
Answer: a. Brudzinski's sign

186. Which condition is characterized by the "buffalo hump" and "moon


facies"?

a. Cushing's syndrome
b. Addison's disease
c. Acromegaly
d. Hyperthyroidism
Answer: a. Cushing's syndrome

187. The preferred imaging modality for evaluating a suspected pulmonary


embolism is:

a. Chest X-ray
b. CT pulmonary angiography
c. MRI
d. Ultrasound
Answer: b. CT pulmonary angiography

188. The classic triad of symptoms for endometriosis includes:

a. Dysmenorrhea, dyspareunia, infertility


b. Menorrhagia, metrorrhagia, pelvic pain
c. Oligomenorrhea, amenorrhea, acne
d. Hypermenorrhea, hot flashes, weight gain
Answer: a. Dysmenorrhea, dyspareunia, infertility

189. The most common cause of acute pancreatitis is:

a. Alcohol consumption
b. Gallstones
c. Hyperlipidemia
d. Trauma
Answer: b. Gallstones

190. In which condition would you expect to find a "whirl sign" on imaging?

a. Intestinal obstruction
b. Appendicitis
c. Pancreatitis
d. Bowel ischemia
Answer: a. Intestinal obstruction

191. Which vitamin is essential for the synthesis of clotting factors?

a. Vitamin A
b. Vitamin C
c. Vitamin D
d. Vitamin K
Answer: d. Vitamin K

192. The first-line treatment for hypertension in a diabetic patient is:

a. Thiazide diuretics
b. ACE inhibitors
c. Calcium channel blockers
d. Beta-blockers
Answer: b. ACE inhibitors

193. A common cause of hypercalcemia is:


a. Hypoparathyroidism
b. Paget's disease
c. Hyperparathyroidism
d. Vitamin D deficiency
Answer: c. Hyperparathyroidism

194. The most common type of inherited hyperlipidemia is:

a. Type I
b. Type IIa
c. Type IIb
d. Type III
Answer: b. Type IIa

195. Which of the following is a classic symptom of asthma?

a. Productive cough
b. Wheezing
c. Hemoptysis
d. Chest pain
Answer: b. Wheezing

196. The standard treatment for anaphylaxis includes:

a. Oral antihistamines
b. Subcutaneous epinephrine
c. Intravenous corticosteroids
d. Intramuscular epinephrine
Answer: d. Intramuscular epinephrine

197. The most common congenital heart defect is:

a. Ventricular septal defect


b. Atrial septal defect
c. Tetralogy of Fallot
d. Coarctation of the aorta
Answer: a. Ventricular septal defect
198. Which test is commonly used to assess liver function?

a. CBC
b. BMP
c. LFT
d. PT/INR
Answer: c. LFT

199. The most effective treatment for a patient with an acute exacerbation of
COPD is:

a. Long-acting beta-agonists
b. Short-acting bronchodilators
c. Inhaled corticosteroids
d. Antibiotics
Answer: b. Short-acting bronchodilators

200. A positive Murphy's sign is indicative of:

a. Appendicitis
b. Cholecystitis
c. Pancreatitis
d. Diverticulitis
Answer: b. Cholecystitis

201. The most common bacterial cause of urinary tract infections in women
is:

a. E. coli
b. Klebsiella
c. Proteus
d. Enterococcus
Answer: a. E. coli

202. Which of the following is a common side effect of corticosteroids?


a. Hypotension
b. Weight loss
c. Osteoporosis
d. Hyperkalemia
Answer: c. Osteoporosis

203. The first-line treatment for obsessive-compulsive disorder (OCD) is:

a. SSRIs
b. CBT
c. Tricyclic antidepressants
d. Benzodiazepines
Answer: a. SSRIs

204. The most common cause of hyperthyroidism is:

a. Hashimoto's thyroiditis
b. Graves' disease
c. Toxic adenoma
d. Subacute thyroiditis
Answer: b. Graves' disease

205. A patient with chronic liver disease is at increased risk for:

a. Pancreatitis
b. Hepatocellular carcinoma
c. Gallstones
d. Cholecystitis
Answer: b. Hepatocellular carcinoma

206. Which of the following is a common complication of diabetes mellitus?

a. Peripheral neuropathy
b. Diabetic retinopathy
c. Diabetic nephropathy
d. All of the above
Answer: d. All of the above
207. The classic symptom of a tension pneumothorax is:

a. Sudden onset chest pain


b. Dyspnea
c. Tracheal deviation
d. Hemoptysis
Answer: c. Tracheal deviation

208. Which of the following is the preferred treatment for acute bacterial
sinusitis?

a. Observation
b. Decongestants
c. Antibiotics
d. Nasal steroids
Answer: c. Antibiotics

209. The main treatment for gout includes:

a. Allopurinol
b. Indomethacin
c. Colchicine
d. Both a and c
Answer: d. Both a and c

210. The most effective way to reduce the risk of cardiovascular disease is:

a. Smoking cessation
b. Weight loss
c. Regular exercise
d. Healthy diet
Answer: a. Smoking cessation

211. The classic presentation of rheumatoid arthritis includes:

a. Asymmetric joint swelling


b. Morning stiffness lasting more than 30 minutes
c. Joint swelling in the DIP joints
d. Osteophyte formation
Answer: b. Morning stiffness lasting more than 30 minutes

212. Which type of insulin is considered a rapid-acting insulin?

a. Insulin glargine
b. Insulin detemir
c. Insulin lispro
d. Insulin NPH
Answer: c. Insulin lispro

213. The first-line treatment for hypertension in the elderly is typically:

a. ACE inhibitors
b. Calcium channel blockers
c. Diuretics
d. Beta-blockers
Answer: c. Diuretics

214. A common complication of untreated hyperthyroidism is:

a. Hypocalcemia
b. Osteoporosis
c. Myxedema
d. Cretinism
Answer: b. Osteoporosis

215. The preferred initial treatment for a patient with stable angina is:

a. Beta-blockers
b. Nitrates
c. Calcium channel blockers
d. Antiplatelet agents
Answer: a. Beta-blockers

216. Which of the following is a key feature of nephrotic syndrome?


a. Hematuria
b. Proteinuria
c. Hypertension
d. Oliguria
Answer: b. Proteinuria

217. The main risk factor for developing type 2 diabetes is:

a. Age
b. Obesity
c. Sedentary lifestyle
d. Family history
Answer: b. Obesity

218. The characteristic "steeple sign" on an X-ray indicates:

a. Croup
b. Epiglottitis
c. Asthma
d. Bronchiolitis
Answer: a. Croup

219. A 45-year-old female presents with fatigue and heavy menstrual


bleeding. The most likely diagnosis is:

a. Hyperthyroidism
b. Uterine fibroids
c. Endometriosis
d. Ovarian cysts
Answer: b. Uterine fibroids

220. The most common cause of secondary hypertension is:

a. Obstructive sleep apnea


b. Hyperaldosteronism
c. Cushing's syndrome
d. Renal artery stenosis
Answer: a. Obstructive sleep apnea

221. The classic triad of symptoms for pheochromocytoma includes:

a. Headaches, palpitations, sweating


b. Palpitations, weight loss, heat intolerance
c. Hypertension, bradycardia, dizziness
d. Fatigue, depression, weight gain
Answer: a. Headaches, palpitations, sweating

222. Which of the following conditions is characterized by a decrease in


anterior pituitary hormones?

a. Cushing's disease
b. Acromegaly
c. Sheehan's syndrome
d. Hyperprolactinemia
Answer: c. Sheehan's syndrome

223. A common side effect of long-term corticosteroid use is:

a. Hyperkalemia
b. Osteoporosis
c. Weight loss
d. Insomnia
Answer: b. Osteoporosis

224. The primary treatment for an acute exacerbation of asthma is:

a. Oral corticosteroids
b. Inhaled corticosteroids
c. Short-acting beta-agonists
d. Long-acting beta-agonists
Answer: c. Short-acting beta-agonists

225. Which of the following is a classic symptom of Crohn's disease?


a. Bloody diarrhea
b. Abdominal pain relieved by defecation
c. Continuous abdominal pain
d. Fistula formation
Answer: d. Fistula formation

226. The most common type of primary liver cancer is:

a. Hepatocellular carcinoma
b. Cholangiocarcinoma
c. Hepatic angiosarcoma
d. Fibrolamellar carcinoma
Answer: a. Hepatocellular carcinoma

227. The mainstay of treatment for peptic ulcer disease caused by H. pylori is:

a. Proton pump inhibitors


b. Antibiotics
c. Antacids
d. Sucralfate
Answer: b. Antibiotics

228. Which electrolyte imbalance is most commonly associated with heart


failure?

a. Hypernatremia
b. Hypokalemia
c. Hyperkalemia
d. Hyponatremia
Answer: d. Hyponatremia

229. The most common cause of acute liver failure in the United States is:

a. Alcohol use
b. Acetaminophen toxicity
c. Viral hepatitis
d. Autoimmune hepatitis
Answer: b. Acetaminophen toxicity

230. A "barking" cough in a child is suggestive of:

a. Asthma
b. Croup
c. Bronchiolitis
d. Pneumonia
Answer: b. Croup

231. The drug of choice for treating hypertension in pregnant women is:

a. Methyldopa
b. Labetalol
c. Nifedipine
d. ACE inhibitors
Answer: a. Methyldopa

232. A common finding in a patient with chronic kidney disease is:

a. Hypocalcemia
b. Hyperphosphatemia
c. Hypokalemia
d. Hypernatremia
Answer: b. Hyperphosphatemia

233. Which of the following is a common cause of dizziness in the elderly?

a. Parkinson's disease
b. Orthostatic hypotension
c. Hypertension
d. Diabetes mellitus
Answer: b. Orthostatic hypotension

234. The preferred imaging study for evaluating a suspected stroke is:
a. MRI
b. CT scan
c. Ultrasound
d. X-ray
Answer: b. CT scan

235. The best initial treatment for a patient with hyperglycemic hyperosmolar
state is:

a. Insulin
b. Fluid resuscitation
c. Electrolyte replacement
d. Oral hypoglycemics
Answer: b. Fluid resuscitation

236. The most common cause of acute sinusitis is:

a. Viral infection
b. Bacterial infection
c. Fungal infection
d. Allergies
Answer: a. Viral infection

237. The primary treatment for a patient diagnosed with chronic obstructive
pulmonary disease (COPD) is:

a. Antibiotics
b. Bronchodilators
c. Corticosteroids
d. Oxygen therapy
Answer: b. Bronchodilators

238. Which of the following is a common symptom of mitral stenosis?

a. Chest pain
b. Atrial fibrillation
c. Syncope
d. Palpitations
Answer: b. Atrial fibrillation

239. The primary indication for a lumbar puncture is:

a. Hypertension
b. Meningitis
c. Stroke
d. Seizures
Answer: b. Meningitis

240. Which condition is associated with a "sausage-shaped" abdomen in


children?

a. Intussusception
b. Appendicitis
c. Volvulus
d. Colonic obstruction
Answer: a. Intussusception

241. he most common cause of secondary amenorrhea is:

a. Pregnancy
b. Polycystic ovary syndrome
c. Thyroid dysfunction
d. Uterine abnormalities
Answer: a. Pregnancy

242. Which of the following drugs is used for the treatment of anaphylaxis?

a. Antihistamines
b. Corticosteroids
c. Epinephrine
d. Beta-blockers
Answer: c. Epinephrine

243. The classic presentation of diabetic ketoacidosis includes:


a. Hyperglycemia, acidosis, and ketosis
b. Hypoglycemia, acidosis, and dehydration
c. Hyperglycemia, alkalosis, and dehydration
d. Hypoglycemia, alkalosis, and ketosis
Answer: a. Hyperglycemia, acidosis, and ketosis

244. The most common pathogen causing bacterial meningitis in adults is:

a. Streptococcus pneumoniae
b. Neisseria meningitidis
c. Listeria monocytogenes
d. Escherichia coli
Answer: a. Streptococcus pneumoniae

245. The first-line treatment for hypertension in patients with diabetes is:

a. Calcium channel blockers


b. Beta-blockers
c. ACE inhibitors
d. Diuretics
Answer: c. ACE inhibitors

246. A patient with a history of asthma presents with wheezing and shortness
of breath. The most appropriate immediate management is:

a. Corticosteroids
b. Long-acting beta-agonists
c. Short-acting beta-agonists
d. Anticholinergics
Answer: c. Short-acting beta-agonists

247. Which of the following is the most common type of thyroid cancer?

a. Medullary thyroid carcinoma


b. Anaplastic thyroid carcinoma
c. Follicular thyroid carcinoma
d. Papillary thyroid carcinoma
Answer: d. Papillary thyroid carcinoma

248. The primary treatment for an acute myocardial infarction (AMI) is:

a. Fibrinolytics
b. Antiplatelet therapy
c. Beta-blockers
d. Statins
Answer: a. Fibrinolytics

249. The most common cause of acute pancreatitis is:

a. Alcohol abuse
b. Gallstones
c. Hyperlipidemia
d. Trauma
Answer: b. Gallstones

250. Which laboratory finding is most characteristic of primary


hyperaldosteronism?

a. Hyperkalemia
b. Hyponatremia
c. Hypokalemia
d. Hypercalcemia
Answer: c. Hypokalemia

251. A patient presents with sudden onset chest pain, shortness of breath, and
hypoxia. The most likely diagnosis is:

a. Pneumothorax
b. Pulmonary embolism
c. Myocardial infarction
d. Aortic dissection
Answer: b. Pulmonary embolism
252. The hallmark of chronic renal failure on laboratory tests is:

a. Decreased serum creatinine


b. Elevated serum creatinine
c. Increased urine output
d. Decreased urine specific gravity
Answer: b. Elevated serum creatinine

253. Which type of diabetes is characterized by autoimmune destruction of


pancreatic beta cells?

a. Type 1 diabetes
b. Type 2 diabetes
c. Gestational diabetes
d. Latent autoimmune diabetes in adults (LADA)
Answer: a. Type 1 diabetes

254. The classic triad of symptoms in a patient with pheochromocytoma


includes:

a. Sweating, palpitations, headache


b. Weight gain, fatigue, depression
c. Nausea, vomiting, abdominal pain
d. Chest pain, dyspnea, cough
Answer: a. Sweating, palpitations, headache

255. Which of the following is a characteristic feature of Crohn's disease?

a. Continuous lesions
b. Skip lesions
c. Presence of pseudopolyps
d. Involvement of the rectum in all cases
Answer: b. Skip lesions

256. The most common type of lung cancer in non-smokers is:


a. Squamous cell carcinoma
b. Adenocarcinoma
c. Small cell lung cancer
d. Large cell carcinoma
Answer: b. Adenocarcinoma

257. The primary treatment for anaphylactic shock includes:

a. Antihistamines
b. Corticosteroids
c. Fluid resuscitation
d. Epinephrine
Answer: d. Epinephrine

258. Which condition is characterized by the presence of a "target" or


"bull's-eye" lesion?

a. Psoriasis
b. Erythema multiforme
c. Dermatitis herpetiformis
d. Acne vulgaris
Answer: b. Erythema multiforme

259. The primary diagnostic test for a suspected myocardial infarction is:

a. ECG
b. Echocardiogram
c. Cardiac biomarkers
d. Chest X-ray
Answer: a. ECG

260. A patient presents with weight loss, fatigue, and diarrhea. The most
likely diagnosis is:

a. Hyperthyroidism
b. Malignancy
c. Inflammatory bowel disease
d. Diabetes mellitus
Answer: c. Inflammatory bowel disease

261. The characteristic "duskiness" of the skin is associated with:

a. Addison's disease
b. Cushing's syndrome
c. Acromegaly
d. Hyperthyroidism
Answer: a. Addison's disease

262. The best initial therapy for a patient with acute heart failure is:

a. Diuretics
b. Beta-blockers
c. ACE inhibitors
d. Digoxin
Answer: a. Diuretics

263. Which of the following conditions is characterized by decreased bone


density?

a. Osteoporosis
b. Osteoarthritis
c. Osteomyelitis
d. Osteosarcoma
Answer: a. Osteoporosis

264. The primary role of vitamin D in the body is to:

a. Promote calcium absorption


b. Increase blood pressure
c. Support immune function
d. Regulate glucose metabolism
Answer: a. Promote calcium absorption

265. A "butterfly rash" on the face is characteristic of:


a. Rosacea
b. Psoriasis
c. Systemic lupus erythematosus
d. Eczema
Answer: c. Systemic lupus erythematosus

266. The classic symptom of glomerulonephritis is:

a. Hematuria
b. Dysuria
c. Oliguria
d. Proteinuria
Answer: a. Hematuria

267. The first-line treatment for major depressive disorder is:

a. Psychotherapy
b. Electroconvulsive therapy
c. Selective serotonin reuptake inhibitors (SSRIs)
d. Tricyclic antidepressants
Answer: c. Selective serotonin reuptake inhibitors (SSRIs)

268. The most common pathogen responsible for community-acquired


pneumonia is:

a. Streptococcus pneumoniae
b. Mycoplasma pneumoniae
c. Haemophilus influenzae
d. Legionella pneumophila
Answer: a. Streptococcus pneumoniae

269. The primary treatment for a patient with hyperlipidemia is:

a. Statins
b. Fibrates
c. Niacin
d. Bile acid sequestrants
Answer: a. Statins

270. The most common cause of iron deficiency anemia in adults is:

a. Chronic blood loss


b. Dietary deficiency
c. Malabsorption
d. Hemolysis
Answer: a. Chronic blood loss

271. The "seagull sign" on ultrasound is indicative of:

a. Cholecystitis
b. Pancreatitis
c. Bowel obstruction
d. Gallstones
Answer: a. Cholecystitis

272. The most common congenital heart defect is:

a. Ventricular septal defect


b. Atrial septal defect
c. Patent ductus arteriosus
d. Coarctation of the aorta
Answer: a. Ventricular septal defect

273. Which of the following is a common side effect of long-term


corticosteroid use?

a. Hypotension
b. Hyperglycemia
c. Hypokalemia
d. Hypocalcemia
Answer: b. Hyperglycemia

274. The primary treatment for acute bacterial sinusitis is:


a. Decongestants
b. Antibiotics
c. Nasal saline irrigation
d. Antihistamines
Answer: b. Antibiotics

275. In diabetes insipidus, the hormone that is deficient is:

a. Insulin
b. Glucagon
c. Antidiuretic hormone (ADH)
d. Aldosterone
Answer: c. Antidiuretic hormone (ADH)

276. Which of the following is a common complication of diabetes mellitus?

a. Hyperkalemia
b. Peripheral neuropathy
c. Thrombocytopenia
d. Hypercalcemia
Answer: b. Peripheral neuropathy

277. The classic triad of symptoms in a patient with aortic stenosis includes:

a. Dyspnea, angina, syncope


b. Fatigue, palpitations, edema
c. Cough, wheezing, hemoptysis
d. Chest pain, fever, malaise
Answer: a. Dyspnea, angina, syncope

278. The most common site for ectopic pregnancy is:

a. Ovaries
b. Uterine cavity
c. Fallopian tubes
d. Cervix
Answer: c. Fallopian tubes
279. Which of the following is the most effective way to prevent the spread of
infections in healthcare settings?

a. Hand hygiene
b. Use of gloves
c. Face masks
d. Isolation
Answer: a. Hand hygiene

280. The primary treatment for hyperthyroidism is:

a. Beta-blockers
b. Radioactive iodine
c. Antithyroid drugs
d. Surgery
Answer: b. Radioactive iodine

281. The presence of anti-dsDNA antibodies is most closely associated with:

a. Rheumatoid arthritis
b. Systemic lupus erythematosus
c. Sjögren's syndrome
d. Scleroderma
Answer: b. Systemic lupus erythematosus

282. The most common cause of hematuria in adults is:

a. Bladder cancer
b. Urinary tract infection
c. Kidney stones
d. Prostate disease
Answer: b. Urinary tract infection

283. In chronic obstructive pulmonary disease (COPD), the primary


mechanism of airflow limitation is:
a. Bronchoconstriction
b. Inflammation and mucus production
c. Alveolar destruction
d. Airway hyperreactivity
Answer: b. Inflammation and mucus production

284. The most sensitive test for the diagnosis of syphilis is:

a. VDRL
b. RPR
c. FTA-ABS
d. Dark field microscopy
Answer: c. FTA-ABS

285. Which of the following is a major risk factor for developing breast
cancer?

a. Low body mass index (BMI)


b. Early menopause
c. Family history of breast cancer
d. Regular physical activity
Answer: c. Family history of breast cancer

286. The first-line treatment for mild to moderate depression is:

a. Antidepressants
b. Cognitive behavioral therapy
c. Electroconvulsive therapy
d. Herbal supplements
Answer: b. Cognitive behavioral therapy

287. Which of the following is a contraindication to the use of aspirin?

a. Hypertension
b. Asthma
c. Active peptic ulcer disease
d. Diabetes
Answer: c. Active peptic ulcer disease

288. The most common cause of cirrhosis in the United States is:

a. Alcoholic liver disease


b. Viral hepatitis
c. Non-alcoholic fatty liver disease
d. Hemochromatosis
Answer: a. Alcoholic liver disease

289. In the context of heart failure, "ejection fraction" refers to:

a. The volume of blood in the heart after contraction


b. The percentage of blood pumped out of the heart with each beat
c. The total volume of blood in the heart
d. The pressure in the left ventricle during diastole
Answer: b. The percentage of blood pumped out of the heart with each beat

290. Which condition is characterized by the presence of a "saddle nose"


deformity?

a. Granulomatosis with polyangiitis


b. Ehlers-Danlos syndrome
c. Osteogenesis imperfecta
d. Marfan syndrome
Answer: a. Granulomatosis with polyangiitis

291. The most common cause of post-operative fever is:

a. Wound infection
b. Atelectasis
c. Urinary tract infection
d. Deep vein thrombosis
Answer: b. Atelectasis

292. The first-line treatment for seasonal allergic rhinitis is:


a. Decongestants
b. Antihistamines
c. Corticosteroids
d. Leukotriene receptor antagonists
Answer: b. Antihistamines

293. A patient presents with jaundice, dark urine, and pale stools. The likely
diagnosis is: a. Hepatitis
b. Hemolytic anemia
c. Cholestasis
d. Cirrhosis
Answer: c. Cholestasis
294. The mainstay of treatment for a patient with asthma is:

a. Long-acting beta-agonists
b. Inhaled corticosteroids
c. Leukotriene receptor antagonists
d. Theophylline
Answer: b. Inhaled corticosteroids

295. Which of the following is the most common complication of diabetes


mellitus?

a. Peripheral vascular disease


b. Retinopathy
c. Neuropathy
d. Nephropathy
Answer: c. Neuropathy

296. The most sensitive test for detecting hyperparathyroidism is:

a. Serum calcium level


b. Serum parathyroid hormone level
c. 24-hour urine calcium excretion
d. Bone density scan
Answer: b. Serum parathyroid hormone level
297. The classic presentation of appendicitis includes:

a. Right upper quadrant pain


b. Left lower quadrant pain
c. Periumbilical pain migrating to the right lower quadrant
d. Epigastric pain
Answer: c. Periumbilical pain migrating to the right lower quadrant

298. Which of the following is a common symptom of hyperthyroidism?

a. Weight gain
b. Cold intolerance
c. Tachycardia
d. Bradycardia
Answer: c. Tachycardia

299. The best initial imaging study for suspected kidney stones is:

a. MRI
b. Ultrasound
c. CT scan without contrast
d. X-ray
Answer: c. CT scan without contrast

300. The most common cause of community-acquired pneumonia in children


is:

a. Streptococcus pneumoniae
b. Mycoplasma pneumoniae
c. Chlamydia pneumoniae
d. Haemophilus influenzae
Answer: a. Streptococcus pneumoniae

301. Which of the following is a contraindication for the use of beta-blockers?

a. Asthma
b. Hypertension
c. Heart failure
d. Angina
Answer: a. Asthma

302. The most common pathogen associated with urinary tract infections in
women is:

a. Escherichia coli
b. Klebsiella pneumoniae
c. Proteus mirabilis
d. Staphylococcus saprophyticus
Answer: a. Escherichia coli

303. A child presents with a "cherry-red spot" on the macula. The likely
diagnosis is:

a. Tay-Sachs disease
b. Gaucher disease
c. Niemann-Pick disease
d. Krabbe disease
Answer: a. Tay-Sachs disease

304. The initial treatment for anaphylaxis includes:

a. Antihistamines
b. Corticosteroids
c. Epinephrine
d. Beta-agonists
Answer: c. Epinephrine

305. Which vitamin deficiency is associated with night blindness?

a. Vitamin A
b. Vitamin B12
c. Vitamin C
d. Vitamin D
Answer: a. Vitamin A
306. The primary function of insulin is to:

a. Increase blood glucose levels


b. Decrease blood glucose levels
c. Promote gluconeogenesis
d. Stimulate glycogenolysis
Answer: b. Decrease blood glucose levels

307. In the Glasgow Coma Scale, the maximum score is:

a. 10
b. 12
c. 14
d. 15
Answer: d. 15

308. The most effective treatment for a patient with an acute migraine attack
is:

a. NSAIDs
b. Opioids
c. Triptans
d. Antidepressants
Answer: c. Triptans

309. Which of the following conditions is most associated with


hyperuricemia?

a. Gout
b. Osteoarthritis
c. Rheumatoid arthritis
d. Psoriatic arthritis
Answer: a. Gout

310. The presence of HBsAg indicates:


a. Acute hepatitis B infection
b. Chronic hepatitis B infection
c. Past hepatitis B infection
d. Vaccination against hepatitis B
Answer: a. Acute hepatitis B infection

311. A patient with a history of chronic alcohol use presents with confusion
and ataxia. The likely diagnosis is:

a. Wernicke's encephalopathy
b. Korsakoff syndrome
c. Hepatic encephalopathy
d. Delirium tremens
Answer: a. Wernicke's encephalopathy

312. The most common cause of thyroid storm is:

a. Hypothyroidism
b. Thyroid surgery
c. Infection
d. Untreated hyperthyroidism
Answer: d. Untreated hyperthyroidism

313. The primary mechanism of action of ACE inhibitors is to:

a. Increase heart rate


b. Vasodilate peripheral blood vessels
c. Decrease cardiac output
d. Increase sodium retention
Answer: b. Vasodilate peripheral blood vessels

314. Which of the following is a classic sign of Cushing's syndrome?

a. Weight loss
b. Hypotension
c. Moon facies
d. Hyperpigmentation
Answer: c. Moon facies

315. The best initial test to diagnose diabetes mellitus is:

a. Fasting blood glucose


b. 2-hour oral glucose tolerance test
c. Random blood glucose
d. Hemoglobin A1c
Answer: a. Fasting blood glucose

316. A patient with severe burns is at high risk for:

a. Hyperkalemia
b. Hypokalemia
c. Hypernatremia
d. Hyponatremia
Answer: d. Hyponatremia

317. The most common site of tuberculosis infection is:

a. Lungs
b. Kidneys
c. Bones
d. Lymph nodes
Answer: a. Lungs

318. Which of the following drugs is commonly used for the management of
hypertension in pregnant women?

a. Lisinopril
b. Amlodipine
c. Methyldopa
d. Atenolol
Answer: c. Methyldopa
319. The classic triad of symptoms in a patient with pheochromocytoma
includes:

a. Headache, palpitations, sweating


b. Abdominal pain, diarrhea, weight loss
c. Hypertension, hypoglycemia, flushing
d. Fatigue, weight gain, cold intolerance
Answer: a. Headache, palpitations, sweating

320. In the context of gastrointestinal bleeding, the term "melena" refers to:

a. Fresh red blood in stool


b. Black, tarry stools
c. Bright red blood per rectum
d. Blood mixed with mucus
Answer: b. Black, tarry stools

321. The most common organism responsible for bacterial meningitis in


adults is:

a. Neisseria meningitidis
b. Streptococcus pneumoniae
c. Listeria monocytogenes
d. Haemophilus influenzae
Answer: b. Streptococcus pneumoniae

322. Which of the following is a typical feature of Parkinson's disease?

a. Hyperactivity
b. Bradykinesia
c. Tremor of the hands only at rest
d. Spasticity
Answer: b. Bradykinesia

323. The primary mechanism of action of warfarin is:


a. Direct thrombin inhibition
b. Factor Xa inhibition
c. Vitamin K antagonism
d. Platelet aggregation inhibition
Answer: c. Vitamin K antagonism

324. A patient with a high serum creatinine and low urine output is likely
experiencing:

a. Post-renal azotemia
b. Pre-renal azotemia
c. Intrinsic renal failure
d. Functional renal failure
Answer: b. Pre-renal azotemia

325. The most effective treatment for a patient with essential hypertension is:

a. Lifestyle modifications
b. Diuretics
c. ACE inhibitors
d. Calcium channel blockers
Answer: a. Lifestyle modifications

326. A patient with fever, weight loss, and cough is likely suffering from:

a. Tuberculosis
b. Pneumonia
c. Chronic bronchitis
d. Lung cancer
Answer: a. Tuberculosis

327. The gold standard for the diagnosis of pulmonary embolism is:

a. Chest X-ray
b. CT pulmonary angiography
c. D-dimer test
d. V/Q scan
Answer: b. CT pulmonary angiography

328. The classic sign of nephrotic syndrome includes:

a. Hematuria
b. Proteinuria
c. Oliguria
d. Hypertension
Answer: b. Proteinuria

329. Which of the following is a common cause of secondary hypertension?

a. Stress
b. Obesity
c. Hyperaldosteronism
d. High salt intake
Answer: c. Hyperaldosteronism

330. The most common cause of hypothyroidism in adults is:

a. Hashimoto's thyroiditis
b. Iodine deficiency
c. Thyroid surgery
d. Radiation therapy
Answer: a. Hashimoto's thyroiditis

331. Which of the following is a risk factor for developing type 2 diabetes?

a. Low BMI
b. Sedentary lifestyle
c. Vegetarian diet
d. Regular exercise
Answer: b. Sedentary lifestyle

332. A common cause of acute pancreatitis is:


a. Alcohol abuse
b. Diabetes mellitus
c. Hyperlipidemia
d. Gallstones
Answer: d. Gallstones

333. The classic triad of symptoms in nephrotic syndrome includes:

a. Hypertension, edema, hematuria


b. Edema, proteinuria, hypoalbuminemia
c. Flank pain, fever, dysuria
d. Hematuria, dysuria, urinary urgency
Answer: b. Edema, proteinuria, hypoalbuminemia

334. The most common type of skin cancer is:

a. Melanoma
b. Squamous cell carcinoma
c. Basal cell carcinoma
d. Merkel cell carcinoma
Answer: c. Basal cell carcinoma

335. The main cause of peptic ulcer disease is:

a. Stress
b. H. pylori infection
c. Excessive NSAID use
d. High-fat diet
Answer: b. H. pylori infection

336. In patients with chronic kidney disease, the most common electrolyte
disturbance is:

a. Hyperkalemia
b. Hypocalcemia
c. Hypernatremia
d. Hypomagnesemia
Answer: a. Hyperkalemia

337. The most sensitive test for diagnosing myocardial infarction is:

a. CK-MB
b. Troponin I
c. Myoglobin
d. LDH
Answer: b. Troponin I

338. Which of the following is a common side effect of long-term


corticosteroid therapy?

a. Hyperkalemia
b. Osteoporosis
c. Hypoglycemia
d. Bradycardia
Answer: b. Osteoporosis

339. The drug of choice for treating acute asthma attacks is:

a. Salmeterol
b. Montelukast
c. Albuterol
d. Beclomethasone
Answer: c. Albuterol

340. A common presentation of hyperparathyroidism is:

a. Hypocalcemia
b. Hypercalcemia
c. Hypophosphatemia
d. Hyperphosphatemia
Answer: b. Hypercalcemia
341. The primary treatment for a patient with atrial fibrillation and rapid
ventricular response is:

a. Digoxin
b. Amiodarone
c. Beta-blockers
d. Anticoagulation
Answer: c. Beta-blockers

342. In patients with chronic obstructive pulmonary disease (COPD), the main
pathological change is:

a. Bronchoconstriction
b. Alveolar destruction
c. Mucous hypersecretion
d. Inflammation
Answer: b. Alveolar destruction

343. The classic symptom of rheumatoid arthritis is:

a. Morning stiffness
b. Joint swelling
c. Joint pain with activity
d. Fever
Answer: a. Morning stiffness

344. A common complication of untreated streptococcal pharyngitis is:

a. Rheumatic fever
b. Infectious mononucleosis
c. Sinusitis
d. Otitis media
Answer: a. Rheumatic fever

345. The main cause of secondary adrenal insufficiency is:


a. Autoimmune destruction
b. Chronic use of glucocorticoids
c. Infection
d. Tumor
Answer: b. Chronic use of glucocorticoids

346. The most common cause of hyperthyroidism is:

a. Graves' disease
b. Thyroid adenoma
c. Toxic multinodular goiter
d. Hashimoto's thyroiditis
Answer: a. Graves' disease

347. The presence of anti-citrullinated protein antibodies (ACPA) is most


specific for:

a. Osteoarthritis
b. Gout
c. Rheumatoid arthritis
d. Psoriatic arthritis
Answer: c. Rheumatoid arthritis

348. The classic symptom of systemic lupus erythematosus (SLE) is:

a. Malar rash
b. Discoid rash
c. Photosensitivity
d. Oral ulcers
Answer: a. Malar rash

349. The most common pathogen causing community-acquired pneumonia


in adults is:

a. Streptococcus pneumoniae
b. Haemophilus influenzae
c. Mycoplasma pneumoniae
d. Chlamydia pneumoniae
Answer: a. Streptococcus pneumoniae

350. The hallmark of congestive heart failure is:

a. Hypotension
b. Peripheral edema
c. S3 gallop
d. Pulmonary hypertension
Answer: c. S3 gallop

351. The most common cause of acute kidney injury is:

a. Pre-renal causes
b. Post-renal causes
c. Intrinsic renal causes
d. Glomerular causes
Answer: a. Pre-renal causes

352. Which medication is commonly used as a first-line treatment for


hypertension?

a. Calcium channel blockers


b. ACE inhibitors
c. Beta-blockers
d. Diuretics
Answer: b. ACE inhibitors

353. The most common location for an ectopic pregnancy is:

a. Ovaries
b. Uterine tube
c. Abdominal cavity
d. Cervix
Answer: b. Uterine tube

354. The most characteristic finding in a patient with multiple sclerosis is:
a. Muscle weakness
b. Spasticity
c. Visual disturbances
d. Sensory loss
Answer: c. Visual disturbances

355. The drug of choice for hypertensive emergencies is:

a. Nitroprusside
b. Labetalol
c. Amlodipine
d. Metoprolol
Answer: a. Nitroprusside

356. A common side effect of amiodarone is:

a. Hepatotoxicity
b. Nephrotoxicity
c. Pulmonary fibrosis
d. Gastrointestinal bleeding
Answer: c. Pulmonary fibrosis

357. In the context of stroke, the term "hemiparesis" refers to:

a. Loss of speech
b. Weakness on one side of the body
c. Loss of vision
d. Numbness
Answer: b. Weakness on one side of the body

358. The most common cause of chronic cough in adults is:

a. Asthma
b. Gastroesophageal reflux disease (GERD)
c. Chronic bronchitis
d. Postnasal drip
Answer: d. Postnasal drip
359. Which test is most useful for diagnosing deep vein thrombosis (DVT)?

a. CT scan
b. Ultrasound
c. D-dimer
d. MRI
Answer: b. Ultrasound

360. The primary treatment for a patient diagnosed with coronary artery
disease is:

a. Surgical intervention
b. Lifestyle modification
c. Pharmacotherapy
d. Coronary artery bypass grafting
Answer: b. Lifestyle modification

361. The most common cause of iron deficiency anemia is:

a. Hemorrhage
b. Malnutrition
c. Hemolysis
d. Bone marrow disorders
Answer: a. Hemorrhage

362. The initial treatment for hypercalcemia includes:

a. Diuretics
b. Bisphosphonates
c. Hydration
d. Calcitonin
Answer: c. Hydration

363. Which of the following is a common symptom of hypothyroidism?

a. Weight loss
b. Tachycardia
c. Cold intolerance
d. Diarrhea
Answer: c. Cold intolerance

364. The classic presentation of appendicitis includes:

a. Right upper quadrant pain


b. Rebound tenderness in the left lower quadrant
c. Periumbilical pain migrating to the right lower quadrant
d. Constipation
Answer: c. Periumbilical pain migrating to the right lower quadrant

365. Which type of diabetes is characterized by the absence of insulin


production?

a. Type 1 diabetes
b. Type 2 diabetes
c. Gestational diabetes
d. Maturity-onset diabetes of the young
Answer: a. Type 1 diabetes

366. The best initial test to evaluate for congestive heart failure is:

a. Echocardiogram
b. Chest X-ray
c. BNP levels
d. Electrocardiogram
Answer: b. Chest X-ray

367. The first-line treatment for hyperlipidemia is:

a. Statins
b. Fibrates
c. Niacin
d. Bile acid sequestrants
Answer: a. Statins
368. The most common cause of bacterial meningitis in adults is:

a. Streptococcus pneumoniae
b. Neisseria meningitidis
c. Listeria monocytogenes
d. Haemophilus influenzae
Answer: a. Streptococcus pneumoniae

369. Which of the following is a characteristic of Crohn's disease?

a. Continuous colonic involvement


b. Fistula formation
c. Predominantly affects the rectum
d. Mucosal surface only
Answer: b. Fistula formation

370. The most common site for a peptic ulcer is:

a. Duodenum
b. Stomach
c. Esophagus
d. Jejunum
Answer: a. Duodenum

371. The mainstay treatment for patients with bipolar disorder is:

a. Antidepressants
b. Mood stabilizers
c. Antipsychotics
d. Anxiolytics
Answer: b. Mood stabilizers

372. The most effective vaccine for preventing influenza is:

a. Inactivated vaccine
b. Live attenuated vaccine
c. Recombinant vaccine
d. DNA vaccine
Answer: a. Inactivated vaccine

373. The most common presenting symptom of chronic kidney disease is:

a. Hematuria
b. Edema
c. Fatigue
d. Hypertension
Answer: c. Fatigue

374. The first-line treatment for anaphylaxis is:

a. Corticosteroids
b. Antihistamines
c. Epinephrine
d. Inhaled bronchodilators
Answer: c. Epinephrine

375. Which of the following is a common feature of systemic sclerosis?

a. Photosensitivity
b. Sclerodactyly
c. Erythema nodosum
d. Raynaud's phenomenon
Answer: b. Sclerodactyly

376. The most common cause of secondary hypertension is:

a. Renal artery stenosis


b. Cushing's syndrome
c. Hyperaldosteronism
d. Pheochromocytoma
Answer: a. Renal artery stenosis

377. The first-line treatment for bacterial vaginosis is:


a. Metronidazole
b. Clindamycin
c. Azithromycin
d. Fluconazole
Answer: a. Metronidazole

378. The classic triad of symptoms in acute aortic dissection includes:

a. Chest pain, back pain, and hypertension


b. Chest pain, dyspnea, and syncope
c. Chest pain, cough, and hemoptysis
d. Chest pain, palpitations, and anxiety
Answer: a. Chest pain, back pain, and hypertension

379. In a patient with heart failure, which of the following medications is


contraindicated?

a. ACE inhibitors
b. Beta-blockers
c. Calcium channel blockers
d. Diuretics
Answer: c. Calcium channel blockers

380. The most common side effect of warfarin is:

a. Hepatotoxicity
b. Nephrotoxicity
c. Hemorrhage
d. Thrombocytopenia
Answer: c. Hemorrhage

381. The primary cause of acute respiratory distress syndrome (ARDS) is:

a. Pneumonia
b. Sepsis
c. Trauma
d. Aspiration
Answer: b. Sepsis

382. Which of the following is a contraindication to thrombolytic therapy in


acute myocardial infarction?

a. History of hypertension
b. Age over 75 years
c. Prior stroke
d. Active bleeding
Answer: d. Active bleeding

383. The most common symptom of congestive heart failure is:

a. Dyspnea
b. Fatigue
c. Edema
d. Orthopnea
Answer: a. Dyspnea

384. Which of the following is a classic finding in asthma?

a. Clubbing of fingers
b. Wheezing
c. Hypercapnia
d. Cyanosis
Answer: b. Wheezing

385. The most common cause of acute renal failure in hospitalized patients is:

a. Prerenal azotemia
b. Acute tubular necrosis
c. Postrenal obstruction
d. Interstitial nephritis
Answer: b. Acute tubular necrosis
386. Which of the following drugs is a selective serotonin reuptake inhibitor
(SSRI)?

a. Amitriptyline
b. Sertraline
c. Phenelzine
d. Bupropion
Answer: b. Sertraline

387. The mainstay of treatment for hypertension in pregnancy is:

a. Labetalol
b. Methyldopa
c. Nifedipine
d. ACE inhibitors
Answer: b. Methyldopa

388. The most common cause of community-acquired pneumonia in children


is:

a. Streptococcus pneumoniae
b. Mycoplasma pneumoniae
c. Chlamydia pneumoniae
d. Respiratory syncytial virus (RSV)
Answer: a. Streptococcus pneumoniae

389. The most effective long-term management for chronic obstructive


pulmonary disease (COPD) is:

a. Inhaled corticosteroids
b. Long-acting bronchodilators
c. Oxygen therapy
d. Smoking cessation
Answer: d. Smoking cessation

390. The first-line treatment for generalized anxiety disorder is:


a. Benzodiazepines
b. Selective serotonin reuptake inhibitors (SSRIs)
c. Cognitive behavioral therapy
d. Buspirone
Answer: b. Selective serotonin reuptake inhibitors (SSRIs)

391. The most common site for metastatic breast cancer is:

a. Liver
b. Bone
c. Lung
d. Brain
Answer: b. Bone

392. The classic symptom of aortic stenosis is:

a. Dyspnea on exertion
b. Syncope
c. Angina
d. Palpitations
Answer: a. Dyspnea on exertion

393. The treatment of choice for anaphylactic shock is:

a. Antihistamines
b. Corticosteroids
c. Epinephrine
d. Oxygen therapy
Answer: c. Epinephrine

394. The most common organism causing urinary tract infections in women
is:

a. Staphylococcus saprophyticus
b. Escherichia coli
c. Klebsiella pneumoniae
d. Proteus mirabilis
Answer: b. Escherichia coli

395. The primary goal of diabetes management is to maintain:

a. Blood pressure
b. Lipid levels
c. Blood glucose levels
d. Weight
Answer: c. Blood glucose levels

396. Which of the following is the most effective treatment for rheumatoid
arthritis?

a. Nonsteroidal anti-inflammatory drugs (NSAIDs)


b. Disease-modifying antirheumatic drugs (DMARDs)
c. Corticosteroids
d. Biologic agents
Answer: b. Disease-modifying antirheumatic drugs (DMARDs)

397. The most common cause of hyperthyroidism is:

a. Hashimoto's thyroiditis
b. Thyroid adenoma
c. Graves' disease
d. Iodine deficiency
Answer: c. Graves' disease

398. The classic triad of symptoms in Addison's disease includes:

a. Hyperpigmentation, fatigue, hypotension


b. Weight gain, hypertension, hypokalemia
c. Hyperglycemia, polydipsia, polyuria
d. Cold intolerance, weight loss, tachycardia
Answer: a. Hyperpigmentation, fatigue, hypotension

399. The preferred initial treatment for chronic pain is:


a. Opioids
b. NSAIDs
c. Acetaminophen
d. Antidepressants
Answer: b. NSAIDs

400. The primary cause of peptic ulcers is:

a. Stress
b. Helicobacter pylori infection
c. Excessive alcohol consumption
d. Smoking
Answer: b. Helicobacter pylori infection

401. The most common cause of acute liver failure in the United States is:

a. Viral hepatitis
b. Alcohol abuse
c. Drug-induced liver injury
d. Autoimmune hepatitis
Answer: c. Drug-induced liver injury

402. The classic symptom of multiple sclerosis is:

a. Intention tremor
b. Dysphagia
c. Hemiparesis
d. Seizures
Answer: a. Intention tremor

403. The initial treatment for congestive heart failure with reduced ejection
fraction includes:

a. Digoxin
b. Loop diuretics
c. Beta-blockers
d. ACE inhibitors
Answer: d. ACE inhibitors

404. The primary cause of hyponatremia in hospitalized patients is:

a. Excessive fluid intake


b. Diuretic use
c. Syndrome of inappropriate antidiuretic hormone secretion (SIADH)
d. Adrenal insufficiency
Answer: c. Syndrome of inappropriate antidiuretic hormone secretion
(SIADH)

405. The most common site for ectopic pregnancy is:

a. Uterus
b. Ovary
c. Fallopian tube
d. Abdominal cavity
Answer: c. Fallopian tube

406. The best diagnostic test for osteoporosis is:

a. X-ray
b. Bone density scan (DEXA)
c. CT scan
d. MRI
Answer: b. Bone density scan (DEXA)

407. The primary treatment for gout is:

a. NSAIDs
b. Allopurinol
c. Colchicine
d. Corticosteroids
Answer: b. Allopurinol
408. The most common organism causing bacterial endocarditis in drug users
is:

a. Staphylococcus aureus
b. Streptococcus viridans
c. Enterococcus
d. HACEK organisms
Answer: a. Staphylococcus aureus

409. The most common presenting symptom of chronic kidney disease is:

a. Hematuria
b. Fatigue
c. Edema
d. Hypertension
Answer: b. Fatigue

410. Which of the following is a complication of diabetes mellitus?

a. Hyperthyroidism
b. Cushing's syndrome
c. Diabetic neuropathy
d. Addison's disease
Answer: c. Diabetic neuropathy

411. The most common cause of secondary amenorrhea is:

a. Pregnancy
b. Polycystic ovary syndrome
c. Thyroid dysfunction
d. Stress
Answer: a. Pregnancy

412. The first-line medication for chronic obstructive pulmonary disease


(COPD) is:
a. Inhaled corticosteroids
b. Long-acting beta agonists
c. Anticholinergics
d. Systemic corticosteroids
Answer: b. Long-acting beta agonists

413. The initial treatment for an acute asthma exacerbation is:

a. Corticosteroids
b. Oxygen therapy
c. Short-acting beta agonists
d. Leukotriene modifiers
Answer: c. Short-acting beta agonists

414. The most common cause of viral gastroenteritis is:

a. Norovirus
b. Rotavirus
c. Adenovirus
d. Astrovirus
Answer: a. Norovirus

415. The classic symptom of Parkinson's disease is:

a. Bradykinesia
b. Hyperactivity
c. Seizures
d. Ataxia
Answer: a. Bradykinesia

416. The most common cause of community-acquired pneumonia in adults


is:

a. Streptococcus pneumoniae
b. Mycoplasma pneumoniae
c. Haemophilus influenzae
d. Legionella pneumophila
Answer: a. Streptococcus pneumoniae

417. The first-line treatment for hypertension in patients with diabetes is:

a. ACE inhibitors
b. Beta-blockers
c. Calcium channel blockers
d. Diuretics
Answer: a. ACE inhibitors

418. The most common type of primary lung cancer is:

a. Squamous cell carcinoma


b. Adenocarcinoma
c. Small cell lung cancer
d. Large cell carcinoma
Answer: b. Adenocarcinoma

419. The first-line treatment for obsessive-compulsive disorder (OCD) is:

a. Exposure and response prevention therapy


b. Selective serotonin reuptake inhibitors (SSRIs)
c. Antipsychotics
d. Benzodiazepines
Answer: b. Selective serotonin reuptake inhibitors (SSRIs)

420. The most common cause of acute pancreatitis is:

a. Alcohol consumption
b. Gallstones
c. Hyperlipidemia
d. Trauma
Answer: b. Gallstones

421. The most common cause of hypercalcemia is:


a. Malignancy
b. Hyperparathyroidism
c. Vitamin D toxicity
d. Chronic kidney disease
Answer: b. Hyperparathyroidism

422. The primary treatment for essential hypertension includes:

a. Lifestyle changes
b. Diuretics
c. Beta-blockers
d. All of the above
Answer: d. All of the above

423. The characteristic symptom of hyperthyroidism is:

a. Weight gain
b. Cold intolerance
c. Heat intolerance
d. Bradycardia
Answer: c. Heat intolerance

424. The drug of choice for status epilepticus is:

a. Phenytoin
b. Valproate
c. Diazepam
d. Carbamazepine
Answer: c. Diazepam

425. The most common cause of chronic liver disease in the developed world
is:

a. Alcoholic liver disease


b. Non-alcoholic fatty liver disease
c. Viral hepatitis
d. Hemochromatosis
Answer: b. Non-alcoholic fatty liver disease

426. The primary function of insulin is to:

a. Increase blood glucose levels


b. Decrease blood glucose levels
c. Promote gluconeogenesis
d. Increase ketogenesis
Answer: b. Decrease blood glucose levels

427. The hallmark of diabetic ketoacidosis is:

a. Hyperglycemia
b. Hypokalemia
c. Metabolic alkalosis
d. Hypernatremia
Answer: a. Hyperglycemia

428. The classic presentation of a patient with an adrenal crisis includes:

a. Hypertension, bradycardia
b. Hypotension, hyperkalemia
c. Tachycardia, hypoglycemia
d. Hypernatremia, dehydration
Answer: b. Hypotension, hyperkalemia

429. The most common cause of meningitis in adults is:

a. Neisseria meningitidis
b. Streptococcus pneumoniae
c. Listeria monocytogenes
d. Haemophilus influenzae
Answer: b. Streptococcus pneumoniae

430. The initial management of acute coronary syndrome includes:


a. Morphine
b. Aspirin
c. Nitroglycerin
d. All of the above
Answer: d. All of the above

431. The most common type of stroke is:

a. Hemorrhagic
b. Ischemic
c. Transient ischemic attack
d. Cryptogenic
Answer: b. Ischemic

432. The main side effect of long-term steroid use is:

a. Hypertension
b. Hyperglycemia
c. Osteoporosis
d. All of the above
Answer: d. All of the above

433. The first-line treatment for hyperlipidemia is:

a. Statins
b. Niacin
c. Fibrates
d. Bile acid sequestrants
Answer: a. Statins

434. The most common inherited bleeding disorder is:

a. Hemophilia A
b. Hemophilia B
c. von Willebrand disease
d. Thrombocytopenia
Answer: c. von Willebrand disease
435. The best initial test for suspected coronary artery disease is:

a. Cardiac catheterization
b. Stress test
c. Chest X-ray
d. ECG
Answer: b. Stress test

436. The most common cause of sudden cardiac arrest in young athletes is:

a. Hypertrophic cardiomyopathy
b. Coronary artery disease
c. Arrhythmogenic right ventricular cardiomyopathy
d. Myocarditis
Answer: a. Hypertrophic cardiomyopathy

437. The classic triad of symptoms for renal artery stenosis includes:

a. Hypertension, hypokalemia, metabolic alkalosis


b. Abdominal pain, flank pain, hematuria
c. Headache, visual changes, confusion
d. Weight loss, fatigue, night sweats
Answer: a. Hypertension, hypokalemia, metabolic alkalosis

438. The most common cause of obstructive jaundice is:

a. Hepatitis
b. Cirrhosis
c. Gallstones
d. Pancreatic cancer
Answer: c. Gallstones

439. The primary purpose of a Pap smear is to screen for:

a. Ovarian cancer
b. Endometrial cancer
c. Cervical cancer
d. Breast cancer
Answer: c. Cervical cancer

440. The most common presenting symptom of colon cancer is:

a. Rectal bleeding
b. Abdominal pain
c. Change in bowel habits
d. Weight loss
Answer: c. Change in bowel habits

441. The primary effect of beta-blockers in heart failure is:

a. Increase heart rate


b. Decrease myocardial oxygen demand
c. Increase cardiac output
d. Decrease preload
Answer: b. Decrease myocardial oxygen demand

442. The most common cause of hypothyroidism is:

a. Hashimoto's thyroiditis
b. Iodine deficiency
c. Thyroid surgery
d. Radiation therapy
Answer: a. Hashimoto's thyroiditis

443. The first-line treatment for acute bronchospasm in asthma is:

a. Long-acting beta-agonists
b. Anticholinergics
c. Short-acting beta-agonists
d. Corticosteroids
Answer: c. Short-acting beta-agonists

444. The most common genetic cause of intellectual disability is:


a. Down syndrome
b. Fragile X syndrome
c. Klinefelter syndrome
d. Turner syndrome
Answer: b. Fragile X syndrome

445. The first-line treatment for major depressive disorder is:

a. SSRIs
b. Tricyclic antidepressants
c. MAO inhibitors
d. Mood stabilizers
Answer: a. SSRIs

446. The most common cause of acquired immunodeficiency syndrome


(AIDS) is:

a. HIV-1
b. HIV-2
c. HTLV-I
d. HTLV-II
Answer: a. HIV-1

447. The most common cause of acute kidney injury in hospitalized patients
is:

a. Prerenal azotemia
b. Intrarenal injury
c. Postrenal obstruction
d. Glomerulonephritis
Answer: a. Prerenal azotemia

448. The first-line treatment for chronic obstructive pulmonary disease


(COPD) exacerbation is:

a. Antibiotics
b. Corticosteroids
c. Bronchodilators
d. All of the above
Answer: d. All of the above

449. The most common form of dementia is:

a. Alzheimer's disease
b. Vascular dementia
c. Lewy body dementia
d. Frontotemporal dementia
Answer: a. Alzheimer's disease

450. The best initial therapy for newly diagnosed type 2 diabetes is:

a. Metformin
b. Insulin
c. Sulfonylureas
d. Thiazolidinediones
Answer: a. Metformin

451. The first-line treatment for allergic rhinitis is:

a. Antihistamines
b. Decongestants
c. Corticosteroids
d. Leukotriene receptor antagonists
Answer: c. Corticosteroids

452. The most common cause of peptic ulcers is:

a. NSAIDs
b. Stress
c. H. pylori infection
d. Alcohol
Answer: c. H. pylori infection

453. The characteristic feature of chronic bronchitis is:


a. Increased airway resistance
b. Decreased lung compliance
c. Dry cough
d. Airway hyperreactivity
Answer: a. Increased airway resistance

454. The best initial treatment for a patient with anaphylaxis is:

a. Antihistamines
b. Epinephrine
c. Corticosteroids
d. Beta-agonists
Answer: b. Epinephrine

455. The most common presentation of a pulmonary embolism is:

a. Cough
b. Hemoptysis
c. Pleuritic chest pain
d. Shortness of breath
Answer: d. Shortness of breath

456. The gold standard for diagnosing diabetes mellitus is:

a. Fasting blood glucose


b. Oral glucose tolerance test
c. A1C test
d. Random blood glucose
Answer: c. A1C test

457. The most common cause of hyperthyroidism is:

a. Thyroiditis
b. Graves' disease
c. Toxic nodular goiter
d. Iatrogenic
Answer: b. Graves' disease
458. The primary pathogen responsible for community-acquired pneumonia
in adults is:

a. Streptococcus pneumoniae
b. Haemophilus influenzae
c. Mycoplasma pneumoniae
d. Legionella pneumophila
Answer: a. Streptococcus pneumoniae

459. The best initial imaging study for suspected biliary obstruction is:

a. Ultrasound
b. CT scan
c. MRI
d. X-ray
Answer: a. Ultrasound

460. The most common cause of hematuria in adults is:

a. Urinary tract infection


b. Kidney stones
c. Bladder cancer
d. Glomerulonephritis
Answer: b. Kidney stones

461. The most effective treatment for severe acne is:

a. Topical retinoids
b. Oral antibiotics
c. Isotretinoin
d. Benzoyl peroxide
Answer: c. Isotretinoin

462. The drug of choice for hypertension in pregnancy is:

a. Methyldopa
b. Labetalol
c. Nifedipine
d. Aspirin
Answer: b. Labetalol

463. The most common side effect of thiazide diuretics is:

a. Hyperkalemia
b. Hyponatremia
c. Hyperglycemia
d. Hypokalemia
Answer: d. Hypokalemia

464. The classic symptom of appendicitis is:

a. Right upper quadrant pain


b. Left lower quadrant pain
c. Periumbilical pain
d. Flank pain
Answer: c. Periumbilical pain

465. The most effective therapy for severe depression is:

a. Psychotherapy
b. Electroconvulsive therapy
c. Antidepressants
d. Herbal remedies
Answer: b. Electroconvulsive therapy

466. The most common complication of a myocardial infarction is:

a. Arrhythmia
b. Heart failure
c. Cardiac rupture
d. Pericarditis
Answer: a. Arrhythmia

467. The initial treatment for a patient with new-onset atrial fibrillation is:
a. Anticoagulation
b. Rate control
c. Rhythm control
d. Cardioversion
Answer: b. Rate control

468. The most common cause of secondary hypertension is:

a. Primary hyperaldosteronism
b. Cushing's syndrome
c. Renal artery stenosis
d. Sleep apnea
Answer: d. Sleep apnea

469. The first-line treatment for generalized anxiety disorder is:

a. Benzodiazepines
b. SSRIs
c. CBT
d. SNRIs
Answer: b. SSRIs

470. The most common cause of hypopituitarism is:

a. Pituitary adenoma
b. Sheehan's syndrome
c. Trauma
d. Infiltrative disease
Answer: a. Pituitary adenoma

471. The gold standard for the diagnosis of sleep apnea is:

a. Pulse oximetry
b. Polysomnography
c. Home sleep study
d. Clinical assessment
Answer: b. Polysomnography
472. The first-line treatment for hypertension in the elderly is:

a. ACE inhibitors
b. Calcium channel blockers
c. Diuretics
d. Beta-blockers
Answer: c. Diuretics

473. The most common cause of primary adrenal insufficiency is:

a. Addison's disease
b. Congenital adrenal hyperplasia
c. Secondary adrenal insufficiency
d. Adrenal tumors
Answer: a. Addison's disease

474. The typical presentation of a patient with aortic dissection includes:

a. Chest pain radiating to the back


b. Sudden onset of severe headache
c. Flank pain and hematuria
d. Abdominal pain with fever
Answer: a. Chest pain radiating to the back

475. The most common viral cause of gastroenteritis in children is:

a. Rotavirus
b. Norovirus
c. Adenovirus
d. Astrovirus
Answer: a. Rotavirus

476. The best test to diagnose celiac disease is:

a. Endoscopy with biopsy


b. Serological testing
c. Stool test
d. Genetic testing
Answer: b. Serological testing

477. The most common side effect of statins is:

a. Rhabdomyolysis
b. Liver enzyme elevation
c. Muscle pain
d. Gastrointestinal upset
Answer: c. Muscle pain

478. The mainstay of treatment for type 1 diabetes is:

a. Oral hypoglycemics
b. Insulin therapy
c. Lifestyle modification
d. Glucagon
Answer: b. Insulin therapy

479. The most common inherited form of cardiomyopathy is:

a. Dilated cardiomyopathy
b. Hypertrophic cardiomyopathy
c. Restrictive cardiomyopathy
d. Arrhythmogenic right ventricular cardiomyopathy
Answer: b. Hypertrophic cardiomyopathy

480. The most common site of hematogenous spread in osteomyelitis in


children is:

a. Spine
b. Long bones
c. Pelvis
d. Skull
Answer: b. Long bones

481. The first-line treatment for hypertension in diabetic patients is:


a. Thiazide diuretics
b. ACE inhibitors
c. Beta-blockers
d. Calcium channel blockers
Answer: b. ACE inhibitors

482. The most common cause of acute pancreatitis is:

a. Alcohol use
b. Gallstones
c. Hyperlipidemia
d. Trauma
Answer: b. Gallstones

483. The classic triad of symptoms in nephrotic syndrome includes:

a. Hematuria, hypertension, edema


b. Edema, proteinuria, hypoalbuminemia
c. Flank pain, fever, nausea
d. Polyuria, polydipsia, weight loss
Answer: b. Edema, proteinuria, hypoalbuminemia

484. The most effective treatment for Crohn's disease is:

a. Corticosteroids
b. Aminosalicylates
c. Immunosuppressants
d. Antibiotics
Answer: c. Immunosuppressants

485. The most common type of lung cancer is:

a. Squamous cell carcinoma


b. Adenocarcinoma
c. Small cell lung cancer
d. Large cell carcinoma
Answer: b. Adenocarcinoma
486. The primary treatment for hyperkalemia is:

a. Calcium gluconate
b. Insulin and glucose
c. Sodium bicarbonate
d. Diuretics
Answer: b. Insulin and glucose

487. The most common form of dementia in older adults is:

a. Vascular dementia
b. Alzheimer's disease
c. Lewy body dementia
d. Frontotemporal dementia
Answer: b. Alzheimer's disease

488. The initial test for suspected heart failure is:

a. Echocardiogram
b. Chest X-ray
c. BNP levels
d. ECG
Answer: b. Chest X-ray

489. The most common inherited bleeding disorder is:

a. Hemophilia A
b. von Willebrand disease
c. Hemophilia B
d. Factor V Leiden
Answer: b. von Willebrand disease

490. The most effective drug for smoking cessation is:

a. Bupropion
b. Varenicline
c. Nicotine replacement therapy
d. Nortriptyline
Answer: b. Varenicline

491. The primary treatment for hypothyroidism is:

a. Levothyroxine
b. Methimazole
c. Radioactive iodine
d. Surgery
Answer: a. Levothyroxine

492. The most common cause of infectious mononucleosis is:

a. Cytomegalovirus
b. Epstein-Barr virus
c. Human immunodeficiency virus
d. Herpes simplex virus
Answer: b. Epstein-Barr virus

493. The main complication of untreated diabetes mellitus is:

a. Hypertension
b. Neuropathy
c. Retinopathy
d. Cardiovascular disease
Answer: d. Cardiovascular disease

494. The most common cause of iron deficiency anemia is:

a. Hemolysis
b. Blood loss
c. Poor dietary intake
d. Malabsorption
Answer: b. Blood loss

495. The drug of choice for treating systemic lupus erythematosus is:
a. Aspirin
b. Hydroxychloroquine
c. Methotrexate
d. Corticosteroids
Answer: b. Hydroxychloroquine

496. The best initial treatment for a patient with active TB is:

a. Isoniazid
b. Rifampicin
c. Pyrazinamide
d. Multi-drug therapy
Answer: d. Multi-drug therapy

497. The most common type of stroke is:

a. Hemorrhagic stroke
b. Ischemic stroke
c. Transient ischemic attack
d. Cryptogenic stroke
Answer: b. Ischemic stroke

498. The primary screening test for breast cancer is:

a. MRI
b. Ultrasound
c. Mammography
d. Biopsy
Answer: c. Mammography

499. The most common complication of diabetes is:

a. Nephropathy
b. Retinopathy
c. Neuropathy
d. Cardiovascular disease
Answer: d. Cardiovascular disease
500. The most sensitive test for detecting a myocardial infarction is:

a. ECG
b. Troponin levels
c. CK-MB levels
d. Myoglobin levels
Answer: b. Troponin levels

501. The most common site for osteosarcoma is:

a. Proximal femur
b. Distal tibia
c. Distal humerus
d. Spine
Answer: a. Proximal femur

502. The primary treatment for acute respiratory distress syndrome (ARDS)
is:

a. Antibiotics
b. Mechanical ventilation
c. Steroids
d. Bronchodilators
Answer: b. Mechanical ventilation

503. The best initial test for suspected deep vein thrombosis (DVT) is:

a. Ultrasound
b. CT scan
c. MRI
d. D-dimer
Answer: a. Ultrasound

504. The classic presentation of a patient with hyperparathyroidism includes:

a. "Stones, bones, groans, and psychiatric overtones"


b. "Wasting and weakness"
c. "Cold intolerance and weight gain"
d. "Increased thirst and urination"
Answer: a. "Stones, bones, groans, and psychiatric overtones"

505. The most common treatment for gout is:

a. Colchicine
b. Allopurinol
c. NSAIDs
d. Corticosteroids
Answer: c. NSAIDs

506. The most common cause of adrenal crisis is:

a. Acute adrenal insufficiency


b. Stress
c. Infection
d. Trauma
Answer: a. Acute adrenal insufficiency

507. The primary treatment for asthma exacerbation is:

a. Corticosteroids
b. Beta-agonists
c. Anticholinergics
d. Leukotriene receptor antagonists
Answer: b. Beta-agonists

508. The initial management for a patient with suspected spinal cord injury is:

a. MRI
b. CT scan
c. Immobilization
d. Surgical intervention
Answer: c. Immobilization

509. The most common psychiatric disorder in the general population is:
a. Depression
b. Anxiety disorder
c. Bipolar disorder
d. Schizophrenia
Answer: b. Anxiety disorder

510. The primary cause of viral hepatitis is:

a. Alcohol consumption
b. Autoimmune disease
c. Infectious agents
d. Drug toxicity
Answer: c. Infectious agents

511. The most common cause of aortic stenosis in the elderly is:

a. Congenital defect
b. Rheumatic fever
c. Degenerative calcific changes
d. Infective endocarditis
Answer: c. Degenerative calcific changes

512. The primary function of the spleen is:

a. Hematopoiesis
b. Filtration of blood
c. Storage of bile
d. Immune response
Answer: b. Filtration of blood

513. The best initial management for anaphylaxis is:

a. Corticosteroids
b. Antihistamines
c. Epinephrine
d. Oxygen therapy
Answer: c. Epinephrine
514. The most common site of gastrointestinal bleeding in peptic ulcer
disease is:

a. Stomach
b. Duodenum
c. Esophagus
d. Jejunum
Answer: b. Duodenum

515. The classic symptom of aortic dissection is: a. Sudden onset of chest pain
b. Dyspnea
c. Fever
d. Nausea
Answer: a. Sudden onset of chest pain
516. The most common cause of secondary hypertension is: a. Kidney disease
b. Obesity
c. Endocrine disorders
d. Sleep apnea
Answer: a. Kidney disease
517. The standard treatment for a patient with chronic obstructive pulmonary
disease (COPD) exacerbation includes: a. Antibiotics and bronchodilators
b. Corticosteroids only
c. Oxygen therapy only
d. Beta-blockers
Answer: a. Antibiotics and bronchodilators
518. The characteristic rash seen in systemic lupus erythematosus (SLE) is: a.
Butterfly rash
b. Maculopapular rash
c. Erythema multiforme
d. Scaly plaques
Answer: a. Butterfly rash
519. The primary test for diagnosing chronic kidney disease (CKD) is:

a. Serum creatinine
b. Urinalysis
c. Glomerular filtration rate (GFR)
d. Ultrasound
Answer: c. Glomerular filtration rate (GFR)

520. The most effective medication for managing hypertension in patients


with heart failure is:

a. Diuretics
b. ACE inhibitors
c. Beta-blockers
d. Calcium channel blockers
Answer: b. ACE inhibitors

521. The primary cause of microcytic anemia is:

a. Vitamin B12 deficiency


b. Iron deficiency
c. Folic acid deficiency
d. Hemolysis
Answer: b. Iron deficiency

522. The most common cause of chronic cough is:

a. Asthma
b. Gastroesophageal reflux disease (GERD)
c. Chronic bronchitis
d. Allergies
Answer: b. Gastroesophageal reflux disease (GERD)

523. The most important laboratory test for diagnosing diabetes mellitus is:

a. Serum glucose
b. HbA1c
c. Urine glucose
d. Insulin levels
Answer: b. HbA1c
524. The classic triad of symptoms in pheochromocytoma includes:

a. Headache, sweating, palpitations


b. Weight loss, heat intolerance, tremors
c. Abdominal pain, diarrhea, flushing
d. Hypertension, tachycardia, anxiety
Answer: a. Headache, sweating, palpitations

525. The first-line treatment for allergic rhinitis is:

a. Oral antihistamines
b. Intranasal corticosteroids
c. Decongestants
d. Saline nasal sprays
Answer: b. Intranasal corticosteroids

526. The most common type of kidney stone is:

a. Calcium oxalate
b. Uric acid
c. Struvite
d. Cystine
Answer: a. Calcium oxalate

527. The initial management for a patient with a suspected myocardial


infarction is:

a. Nitroglycerin
b. Aspirin
c. Morphine
d. Beta-blockers
Answer: b. Aspirin

528. The first-line treatment for depression is:

a. SSRIs
b. SNRIs
c. TCAs
d. MAOIs
Answer: a. SSRIs

529. The classic sign of appendicitis is:

a. Rebound tenderness
b. McBurney's point tenderness
c. Rigidity
d. Guarding
Answer: b. McBurney's point tenderness

530. The best initial test for diagnosing pneumonia is:

a. Chest X-ray
b. CT scan
c. Sputum culture
d. Bronchoscopy
Answer: a. Chest X-ray

531. The most common cause of hypercalcemia in an outpatient setting is:

a. Malignancy
b. Primary hyperparathyroidism
c. Vitamin D intoxication
d. Sarcoidosis
Answer: b. Primary hyperparathyroidism

532. The drug of choice for managing status epilepticus is:

a. Phenytoin
b. Benzodiazepines
c. Carbamazepine
d. Valproate
Answer: b. Benzodiazepines

533. The primary mechanism of action of ACE inhibitors is:


a. Vasodilation
b. Decreased heart rate
c. Increased renal blood flow
d. Reduced blood volume
Answer: a. Vasodilation

534. The classic feature of Parkinson's disease includes:

a. Tremor at rest
b. Muscle weakness
c. Visual hallucinations
d. Rapid eye movement sleep disorder
Answer: a. Tremor at rest

535. The first-line treatment for allergic reactions is:

a. Corticosteroids
b. Epinephrine
c. Antihistamines
d. Saline nasal sprays
Answer: b. Epinephrine

536. The most common symptom of diabetic neuropathy is:

a. Pain
b. Numbness
c. Weakness
d. Tingling
Answer: a. Pain

537. The classic presentation of hypothyroidism includes:

a. Weight gain, fatigue, cold intolerance


b. Weight loss, heat intolerance, anxiety
c. Palpitations, tremors, sweating
d. Diarrhea, flushing, fever
Answer: a. Weight gain, fatigue, cold intolerance
538. The most common cause of secondary amenorrhea is:

a. Pregnancy
b. Polycystic ovary syndrome (PCOS)
c. Thyroid disorders
d. Stress
Answer: a. Pregnancy

539. The primary test for diagnosing rheumatoid arthritis is:

a. ESR
b. CRP
c. Anti-CCP antibodies
d. Rheumatoid factor
Answer: c. Anti-CCP antibodies

540. The classic triad of symptoms in acute cholecystitis is:

a. Fever, jaundice, abdominal pain


b. Fever, nausea, right upper quadrant pain
c. Jaundice, vomiting, diarrhea
d. Abdominal pain, weight loss, anorexia
Answer: b. Fever, nausea, right upper quadrant pain

541. The most common pathogen causing community-acquired pneumonia


in adults is:

a. Staphylococcus aureus
b. Haemophilus influenzae
c. Streptococcus pneumoniae
d. Mycoplasma pneumoniae
Answer: c. Streptococcus pneumoniae

542. The primary treatment for hyperthyroidism is:

a. Radioactive iodine
b. Surgery
c. Antithyroid medications
d. Beta-blockers
Answer: a. Radioactive iodine

543. The hallmark of chronic heart failure is:

a. Hypertension
b. Dyspnea on exertion
c. Tachycardia
d. Peripheral edema
Answer: b. Dyspnea on exertion

544. The most common type of skin cancer is:

a. Basal cell carcinoma


b. Squamous cell carcinoma
c. Melanoma
d. Merkel cell carcinoma
Answer: a. Basal cell carcinoma

545. The first-line treatment for gout is:

a. Allopurinol
b. Colchicine
c. NSAIDs
d. Corticosteroids
Answer: c. NSAIDs

546. The most reliable indicator of pulmonary function is:

a. FEV1
b. FVC
c. FEV1/FVC ratio
d. PEFR
Answer: c. FEV1/FVC ratio

547. The main side effect of statins is:


a. Myopathy
b. Hepatotoxicity
c. Gastrointestinal upset
d. Insomnia
Answer: a. Myopathy

548. The most common type of stroke is:

a. Hemorrhagic
b. Ischemic
c. Transient ischemic attack
d. Subarachnoid hemorrhage
Answer: b. Ischemic

549. The drug of choice for treating active tuberculosis is:

a. Isoniazid
b. Rifampicin
c. Pyrazinamide
d. Ethambutol
Answer: b. Rifampicin

550. The primary cause of primary hyperparathyroidism is:

a. Parathyroid adenoma
b. Hyperplasia
c. Carcinoma
d. Genetic syndromes
Answer: a. Parathyroid adenoma

551. The classic triad of symptoms in nephrotic syndrome includes:

a. Hematuria, hypertension, proteinuria


b. Edema, proteinuria, hypoalbuminemia
c. Hematuria, flank pain, fever
d. Oliguria, hyperkalemia, acidosis
Answer: b. Edema, proteinuria, hypoalbuminemia
552. The most common cause of hypothyroidism in the developed world is:

a. Autoimmune thyroiditis (Hashimoto's)


b. Iodine deficiency
c. Thyroidectomy
d. Radiation therapy
Answer: a. Autoimmune thyroiditis (Hashimoto's)

553. The primary imaging modality for assessing gallbladder disease is:

a. CT scan
b. MRI
c. Ultrasound
d. X-ray
Answer: c. Ultrasound

554. The first-line treatment for major depressive disorder is:

a. CBT (Cognitive Behavioral Therapy)


b. SSRIs (Selective Serotonin Reuptake Inhibitors)
c. SNRIs (Serotonin-Norepinephrine Reuptake Inhibitors)
d. MAOIs (Monoamine Oxidase Inhibitors)
Answer: b. SSRIs (Selective Serotonin Reuptake Inhibitors)

555. The classic sign of diabetic retinopathy is:

a. Cotton wool spots


b. Hard exudates
c. Retinal hemorrhages
d. All of the above
Answer: d. All of the above

556. The best initial test for suspected adrenal insufficiency is:

a. Serum cortisol
b. ACTH stimulation test
c. Serum aldosterone
d. Dexamethasone suppression test
Answer: a. Serum cortisol

557. The first-line treatment for acute coronary syndrome is:

a. Aspirin
b. Clopidogrel
c. Statins
d. Nitroglycerin
Answer: a. Aspirin

558. The primary mechanism of action of beta-blockers is:

a. Vasodilation
b. Decreased heart rate and contractility
c. Diuresis
d. Reduced renin release
Answer: b. Decreased heart rate and contractility

559. The mainstay of treatment for asthma exacerbation is:

a. Long-acting beta agonists


b. Inhaled corticosteroids
c. Short-acting beta agonists
d. Anticholinergics
Answer: c. Short-acting beta agonists

560. The most common complication of cirrhosis is:

a. Hepatic encephalopathy
b. Ascites
c. Portal hypertension
d. Hepatocellular carcinoma
Answer: b. Ascites

561. The drug of choice for Parkinson's disease is:


a. Levodopa
b. Amantadine
c. Dopamine agonists
d. MAO-B inhibitors
Answer: a. Levodopa

562. The classic sign of appendicitis is:

a. Psoas sign
b. Murphy's sign
c. Rovsing's sign
d. McBurney's point tenderness
Answer: d. McBurney's point tenderness

563. The most common cause of iron deficiency anemia in adults is:

a. Chronic blood loss


b. Diet
c. Malabsorption
d. Hemolysis
Answer: a. Chronic blood loss

564. The first-line treatment for a panic attack is:

a. SSRIs
b. Benzodiazepines
c. CBT (Cognitive Behavioral Therapy)
d. MAOIs
Answer: b. Benzodiazepines

565. The most common cause of an elevated alkaline phosphatase level is:

a. Liver disease
b. Bone disease
c. Cholestasis
d. Pregnancy
Answer: b. Bone disease
566. The most common congenital heart defect is:

a. Ventricular septal defect


b. Atrial septal defect
c. Patent ductus arteriosus
d. Coarctation of the aorta
Answer: a. Ventricular septal defect

567. The primary test to diagnose pulmonary embolism is:

a. Chest X-ray
b. CT pulmonary angiography
c. D-dimer
d. Ventilation-perfusion (V/Q) scan
Answer: b. CT pulmonary angiography

568. The first-line treatment for hypertension in pregnancy is:

a. Methyldopa
b. Labetalol
c. Nifedipine
d. Hydralazine
Answer: b. Labetalol

569. The most common organism causing urinary tract infections is:

a. Escherichia coli
b. Klebsiella pneumoniae
c. Enterococcus faecalis
d. Staphylococcus saprophyticus
Answer: a. Escherichia coli

570. The drug of choice for treating an acute asthma attack is:

a. Inhaled corticosteroids
b. Long-acting beta agonists
c. Short-acting beta agonists
d. Anticholinergics
Answer: c. Short-acting beta agonists

571. The most common cause of secondary hypertension is:

a. Hyperthyroidism
b. Cushing's syndrome
c. Renal artery stenosis
d. Pheochromocytoma
Answer: c. Renal artery stenosis

572. The primary treatment for a stable angina is:

a. Calcium channel blockers


b. Beta-blockers
c. Nitrates
d. Aspirin
Answer: b. Beta-blockers

573. The classic presentation of a myocardial infarction includes:

a. Sharp chest pain radiating to the back


b. Severe chest pain radiating to the left arm
c. Dull pain localized to the epigastrium
d. Pain relieved by sitting forward
Answer: b. Severe chest pain radiating to the left arm

574. The initial imaging study for suspected fractures is:

a. MRI
b. CT scan
c. X-ray
d. Ultrasound
Answer: c. X-ray

575. The first-line treatment for rheumatoid arthritis is:


a. NSAIDs
b. DMARDs (Disease-Modifying Anti-Rheumatic Drugs)
c. Corticosteroids
d. Biologics
Answer: b. DMARDs (Disease-Modifying Anti-Rheumatic Drugs)

576. The most sensitive test for diagnosing myocardial infarction is:

a. CK-MB
b. Troponin
c. ECG
d. Myoglobin
Answer: b. Troponin

577. The most common cause of bacterial meningitis in adults is:

a. Streptococcus pneumoniae
b. Neisseria meningitidis
c. Listeria monocytogenes
d. Haemophilus influenzae
Answer: a. Streptococcus pneumoniae

578. The drug of choice for severe anaphylaxis is:

a. Antihistamines
b. Corticosteroids
c. Epinephrine
d. Oxygen
Answer: c. Epinephrine

579. The main cause of acute pancreatitis is:

a. Gallstones
b. Alcohol consumption
c. Hyperlipidemia
d. Medications
Answer: a. Gallstones
580. The most common symptom of chronic kidney disease is:

a. Hematuria
b. Proteinuria
c. Fatigue
d. Edema
Answer: c. Fatigue

581. The classic triad of symptoms in nephritic syndrome includes:

a. Hematuria, hypertension, and edema


b. Edema, proteinuria, and hypoalbuminemia
c. Hyperkalemia, acidosis, and oliguria
d. Fever, rash, and joint pain
Answer: a. Hematuria, hypertension, and edema

582. The most common cause of hypercalcemia in outpatient settings is:

a. Malignancy
b. Primary hyperparathyroidism
c. Vitamin D intoxication
d. Granulomatous diseases
Answer: b. Primary hyperparathyroidism

583. The standard treatment for hyperlipidemia includes:

a. Statins
b. Niacin
c. Fibrates
d. Omega-3 fatty acids
Answer: a. Statins

584. The most common electrolyte abnormality in hospitalized patients is:

a. Hyperkalemia
b. Hypocalcemia
c. Hyponatremia
d. Hypomagnesemia
Answer: c. Hyponatremia

585. The characteristic skin finding in psoriasis is:

a. Maculopapular rash
b. Erythematous plaques with silvery scales
c. Vesicular lesions
d. Petechiae
Answer: b. Erythematous plaques with silvery scales

586. The primary treatment for peptic ulcer disease is:

a. H2-receptor antagonists
b. Proton pump inhibitors
c. Antacids
d. Sucralfate
Answer: b. Proton pump inhibitors

587. The best initial test for suspected heart failure is:

a. Echocardiogram
b. BNP (B-type Natriuretic Peptide)
c. Chest X-ray
d. EKG
Answer: b. BNP (B-type Natriuretic Peptide)

588. The most common cause of acute bronchitis is:

a. Bacterial infection
b. Viral infection
c. Allergens
d. Environmental pollutants
Answer: b. Viral infection

589. The most common cause of secondary amenorrhea is:


a. Pregnancy
b. Polycystic ovary syndrome (PCOS)
c. Thyroid dysfunction
d. Stress
Answer: a. Pregnancy

590. The primary treatment for bipolar disorder is:

a. SSRIs
b. Mood stabilizers
c. Antipsychotics
d. Benzodiazepines
Answer: b. Mood stabilizers

591. The classic finding in acute cholecystitis is:

a. Murphy's sign
b. Rebound tenderness
c. Guarding
d. Abdominal distension
Answer: a. Murphy's sign

592. The most common cause of hypothyroidism worldwide is:

a. Autoimmune thyroiditis
b. Iodine deficiency
c. Surgical removal of the thyroid
d. Radiation exposure
Answer: b. Iodine deficiency

593. The most common initial manifestation of diabetic neuropathy is:

a. Autonomic dysfunction
b. Painful neuropathy
c. Loss of proprioception
d. Distal symmetric polyneuropathy
Answer: d. Distal symmetric polyneuropathy
594. The best initial therapy for a hypertensive emergency is:

a. Oral antihypertensives
b. Intravenous nitroglycerin
c. Oral beta-blockers
d. Dihydropyridine calcium channel blockers
Answer: b. Intravenous nitroglycerin

595. The most common cause of chronic cough in adults is:

a. Asthma
b. COPD
c. Gastroesophageal reflux disease (GERD)
d. Allergies
Answer: c. Gastroesophageal reflux disease (GERD)

596. The primary risk factor for developing coronary artery disease is:

a. Diabetes
b. Smoking
c. Hypertension
d. Hyperlipidemia
Answer: b. Smoking

597. The initial test for suspected celiac disease is:

a. Serum IgA endomysial antibodies


b. Serum transglutaminase antibodies
c. Small bowel biopsy
d. Genetic testing
Answer: b. Serum transglutaminase antibodies

598. The most common type of thyroid carcinoma is:

a. Papillary carcinoma
b. Follicular carcinoma
c. Medullary carcinoma
d. Anaplastic carcinoma
Answer: a. Papillary carcinoma

599. The first-line treatment for obsessive-compulsive disorder is:

a. SSRIs
b. CBT (Cognitive Behavioral Therapy)
c. Benzodiazepines
d. Antipsychotics
Answer: a. SSRIs

600. The classic triad of symptoms in nephrotic syndrome includes:

a. Hematuria, proteinuria, and hypertension


b. Proteinuria, hypoalbuminemia, and edema
c. Oliguria, hypertension, and hematuria
d. Oliguria, edema, and hyperkalemia
Answer: b. Proteinuria, hypoalbuminemia, and edema

601. The most common cause of acute respiratory distress syndrome (ARDS)
is: a. Pneumonia
b. Sepsis
c. Aspiration
d. Trauma
Answer: b. Sepsis
602. The primary treatment for allergic rhinitis is:

a. Antihistamines
b. Corticosteroids
c. Leukotriene receptor antagonists
d. Decongestants
Answer: b. Corticosteroids

603. The most common presentation of hyperthyroidism is:

a. Weight gain
b. Fatigue
c. Palpitations
d. Cold intolerance
Answer: c. Palpitations

604. The initial management for anaphylaxis includes:

a. Antihistamines
b. Corticosteroids
c. Epinephrine
d. Oxygen therapy
Answer: c. Epinephrine

605. The primary imaging study for suspected pulmonary embolism is:

a. Chest X-ray
b. CT pulmonary angiography
c. Ventilation-perfusion scan
d. MRI
Answer: b. CT pulmonary angiography

606. The characteristic skin finding in chickenpox is

a. Vesicular rash
b. Maculopapular rash
c. Ulcerative lesions
d. Petechiae
Answer: a. Vesicular rash

607. The most common type of lung cancer in smokers is:

a. Squamous cell carcinoma


b. Adenocarcinoma
c. Small cell carcinoma
d. Large cell carcinoma
Answer: c. Small cell carcinoma

608. The primary goal in managing hypertension is to reduce the risk of:
a. Stroke
b. Heart failure
c. Kidney disease
d. All of the above
Answer: d. All of the above

609. The best initial therapy for a patient with acute gout is

a. Colchicine
b. NSAIDs
c. Corticosteroids
d. Allopurinol
Answer: b. NSAIDs

610. The most common side effect of ACE inhibitors is:

a. Hyperkalemia
b. Cough
c. Angioedema
d. Rash
Answer: b. Cough

611. The classic triad of symptoms in multiple myeloma includes:

a. Anemia, bone pain, and hypercalcemia


b. Fatigue, fever, and splenomegaly
c. Hematuria, proteinuria, and edema
d. Weight loss, night sweats, and lymphadenopathy
Answer: a. Anemia, bone pain, and hypercalcemia

612. The first-line treatment for type 2 diabetes mellitus is:

a. Metformin
b. Sulfonylureas
c. Insulin
d. Thiazolidinediones
Answer: a. Metformin
613. The most common cause of iron deficiency anemia is:

a. Chronic disease
b. Nutritional deficiency
c. Blood loss
d. Hemolysis
Answer: c. Blood loss

614. The primary risk factor for developing venous thromboembolism is:

a. Obesity
b. Immobility
c. Smoking
d. Recent surgery
Answer: b. Immobility

615. The classic finding in aortic stenosis is:

a. Harsh systolic ejection murmur


b. Diastolic rumble
c. Systolic murmur with radiation to the left shoulder
d. Opening snap
Answer: a. Harsh systolic ejection murmur

616. The most common complication of diabetes mellitus is:

a. Neuropathy
b. Retinopathy
c. Nephropathy
d. Cardiovascular disease
Answer: d. Cardiovascular disease

617. The treatment of choice for bacterial sinusitis is:

a. Decongestants
b. Antihistamines
c. Antibiotics
d. Corticosteroids
Answer: c. Antibiotics

618. The classic presentation of a pulmonary embolism includes:

a. Hemoptysis, pleuritic chest pain, and dyspnea


b. Cough, fever, and wheezing
c. Chest tightness and reflux
d. Sudden onset of cough and fever
Answer: a. Hemoptysis, pleuritic chest pain, and dyspnea

619. The best initial imaging study for suspected appendicitis is:

a. MRI
b. Ultrasound
c. CT scan
d. X-ray
Answer: b. Ultrasound

620. The most common cause of acute kidney injury is:

a. Pre-renal causes
b. Intrinsic renal causes
c. Post-renal causes
d. Acute tubular necrosis
Answer: a. Pre-renal causes

621. The standard treatment for hypertension in pregnancy is:

a. Methyldopa
b. Labetalol
c. Nifedipine
d. All of the above
Answer: d. All of the above

622. The classic triad of symptoms in Cushing's syndrome includes:


a. Obesity, hypertension, and hypokalemia
b. Weight loss, hyperglycemia, and hirsutism
c. Fatigue, hyperpigmentation, and hypotension
d. Hypercalcemia, bone pain, and renal stones
Answer: a. Obesity, hypertension, and hypokalemia

623. The first-line treatment for asthma exacerbation is:

a. Corticosteroids
b. Short-acting beta-agonists
c. Long-acting beta-agonists
d. Anticholinergics
Answer: b. Short-acting beta-agonists

624. The most common cause of chronic cough in non-smokers is:

a. Asthma
b. GERD
c. Post-nasal drip
d. COPD
Answer: b. GERD

625. The best test for diagnosing tuberculosis is:

a. Chest X-ray
b. Sputum acid-fast bacilli (AFB) smear
c. Interferon gamma release assays
d. Tuberculin skin test
Answer: b. Sputum acid-fast bacilli (AFB) smear

626. The most effective treatment for allergic asthma is:

a. Corticosteroids
b. Beta-agonists
c. Leukotriene modifiers
d. Antihistamines
Answer: a. Corticosteroids
627. The primary goal of cholesterol-lowering therapy is to reduce the risk of:

a. Diabetes
b. Liver disease
c. Cardiovascular events
d. Hypertension
Answer: c. Cardiovascular events

628. The most common complication of diabetes insipidus is:

a. Hypernatremia
b. Hyponatremia
c. Hyperkalemia
d. Hypocalcemia
Answer: a. Hypernatremia

629. The best initial therapy for a patient with heart failure and reduced
ejection fraction is:

a. Diuretics
b. ACE inhibitors
c. Beta-blockers
d. Digoxin
Answer: b. ACE inhibitors

630. The most common cause of pneumonia in adults is: a. Streptococcus


pneumoniae
b. Haemophilus influenzae
c. Mycoplasma pneumoniae
d. Legionella pneumophila
Answer: a. Streptococcus pneumoniae

631. The most effective first-line treatment for major depressive disorder is:
a. Cognitive Behavioral Therapy
b. Selective Serotonin Reuptake Inhibitors (SSRIs)
c. Tricyclic Antidepressants
d. Electroconvulsive Therapy
Answer: b. Selective Serotonin Reuptake Inhibitors (SSRIs)

632. The most common cause of hypercalcemia in outpatient settings is:

a. Malignancy
b. Primary hyperparathyroidism
c. Vitamin D intoxication
d. Sarcoidosis
Answer: b. Primary hyperparathyroidism

633. The classic presentation of appendicitis includes:

a. Left lower quadrant pain


b. Right upper quadrant pain
c. Periumbilical pain that migrates to the right lower quadrant
d. Lower back pain
Answer: c. Periumbilical pain that migrates to the right lower quadrant

634. The primary treatment for hypothyroidism is:

a. Methimazole
b. Levothyroxine
c. Liothyronine
d. Iodine supplementation
Answer: b. Levothyroxine

635. The most common cause of acute pancreatitis is:

a. Alcohol consumption
b. Gallstones
c. Hyperlipidemia
d. Medications
Answer: b. Gallstones
636. The classic triad of symptoms in meningitis includes:

a. Fever, neck stiffness, and altered mental status


b. Fever, headache, and rash
c. Nausea, vomiting, and diarrhea
d. Cough, wheezing, and dyspnea
Answer: a. Fever, neck stiffness, and altered mental status

637. The most common cause of chronic liver disease worldwide is:

a. Alcoholic liver disease


b. Hepatitis B virus infection
c. Hepatitis C virus infection
d. Non-alcoholic fatty liver disease
Answer: c. Hepatitis C virus infection

638. The best test for diagnosing celiac disease is:

a. Serum IgA tissue transglutaminase antibodies


b. Endoscopy with biopsy
c. Serum IgG gliadin antibodies
d. Stool test for fat malabsorption
Answer: a. Serum IgA tissue transglutaminase antibodies

639. The first-line treatment for generalized anxiety disorder is:

a. SSRIs
b. Benzodiazepines
c. Beta-blockers
d. Buspirone
Answer: a. SSRIs

640. The most common type of dementia is:

a. Vascular dementia
b. Lewy body dementia
c. Alzheimer's disease
d. Frontotemporal dementia
Answer: c. Alzheimer's disease

641. The primary imaging modality for detecting fractures is:

a. MRI
b. CT scan
c. X-ray
d. Ultrasound
Answer: c. X-ray

642. The treatment of choice for peptic ulcer disease caused by H. pylori is:

a. Proton pump inhibitors


b. Antibiotics
c. Antacids
d. Bismuth compounds
Answer: b. Antibiotics

643. The most common cause of secondary hypertension is:

a. Chronic kidney disease


b. Obesity
c. Sleep apnea
d. Hyperaldosteronism
Answer: a. Chronic kidney disease

644. The first-line medication for treating hypertension in pregnant women


is:

a. Labetalol
b. Amlodipine
c. Hydrochlorothiazide
d. Methyldopa
Answer: d. Methyldopa

645. The primary diagnostic tool for sleep apnea is:


a. Polysomnography
b. CT scan
c. MRI
d. Sleep diary
Answer: a. Polysomnography

646. The first-line treatment for acute migraine attacks is:

a. NSAIDs
b. Opioids
c. Triptans
d. Antidepressants
Answer: c. Triptans

647. The most common cause of community-acquired pneumonia is:

a. Streptococcus pneumoniae
b. Haemophilus influenzae
c. Mycoplasma pneumoniae
d. Chlamydia pneumoniae
Answer: a. Streptococcus pneumoniae

648. The classic sign of left-sided heart failure is:

a. Peripheral edema
b. Jugular venous distension
c. Pulmonary congestion
d. Ascites
Answer: c. Pulmonary congestion

649. The primary complication of untreated diabetes mellitus is:

a. Hypoglycemia
b. Hypertension
c. Diabetic neuropathy
d. Heart failure
Answer: c. Diabetic neuropathy
650. The most sensitive test for detecting myocardial infarction is:

a. Troponin
b. CK-MB
c. Myoglobin
d. ECG changes
Answer: a. Troponin

651. The first-line treatment for osteoarthritis is:

a. NSAIDs
b. Corticosteroids
c. Disease-modifying antirheumatic drugs (DMARDs)
d. Physical therapy
Answer: a. NSAIDs

652. The classic symptom of hyperthyroidism is:

a. Fatigue
b. Weight gain
c. Heat intolerance
d. Cold intolerance
Answer: c. Heat intolerance

653. The most common cause of aortic regurgitation is:

a. Aortic dissection
b. Rheumatic heart disease
c. Infective endocarditis
d. Aortic dilation
Answer: d. Aortic dilation

654. The primary prevention strategy for coronary artery disease includes:

a. Aspirin therapy
b. Lipid management
c. Lifestyle modification
d. Beta-blockers
Answer: c. Lifestyle modification

655. The best initial therapy for a patient with acute exacerbation of COPD is:

a. Corticosteroids
b. Bronchodilators
c. Antibiotics
d. Oxygen therapy
Answer: b. Bronchodilators

656. The most common cause of stridor in children is:

a. Croup
b. Epiglottitis
c. Foreign body aspiration
d. Laryngomalacia
Answer: a. Croup

657. The standard treatment for hypertension in a patient with heart failure
is:

a. ACE inhibitors
b. Calcium channel blockers
c. Diuretics
d. Beta-blockers
Answer: a. ACE inhibitors

658. The characteristic finding in osteomyelitis is:

a. Joint effusion
b. Bone tenderness and swelling
c. Erythema and warmth
d. All of the above
Answer: d. All of the above

659. The most common type of thyroid cancer is:


a. Papillary carcinoma
b. Follicular carcinoma
c. Medullary carcinoma
d. Anaplastic carcinoma
Answer: a. Papillary carcinoma

660. The primary treatment for peptic ulcer disease associated with NSAID
use is:

a. Antacids
b. Proton pump inhibitors
c. H2 receptor antagonists
d. Sucralfate
Answer: b. Proton pump inhibitors

661. The classic symptom of acute cholecystitis is:

a. Right upper quadrant pain


b. Left lower quadrant pain
c. Epigastric pain
d. Back pain
Answer: a. Right upper quadrant pain

662. The most common organism responsible for urinary tract infections in
women is:

a. Staphylococcus saprophyticus
b. E. coli
c. Enterococcus
d. Klebsiella
Answer: b. E. coli

663. The primary imaging modality for diagnosing gallstones is:

a. CT scan
b. Ultrasound
c. MRI
d. X-ray
Answer: b. Ultrasound

664. The most common cause of viral hepatitis worldwide is:

a. Hepatitis A virus
b. Hepatitis B virus
c. Hepatitis C virus
d. Hepatitis E virus
Answer: b. Hepatitis B virus

665. The best initial treatment for acute asthma exacerbation is:

a. Inhaled corticosteroids
b. Systemic corticosteroids
c. Short-acting beta-agonists
d. Long-acting beta-agonists
Answer: c. Short-acting beta-agonists

666. The primary mechanism of action of ACE inhibitors is:

a. Inhibition of angiotensin II production


b. Blockade of calcium channels
c. Diuretic effect
d. Beta-adrenergic blockade
Answer: a. Inhibition of angiotensin II production

667. The classic sign of rheumatoid arthritis is:

a. Morning stiffness
b. Joint swelling
c. Skin rash
d. Fever
Answer: a. Morning stiffness

668. The most common cause of septic arthritis in adults is:


a. Neisseria gonorrhoeae
b. Staphylococcus aureus
c. Streptococcus pneumoniae
d. Mycobacterium tuberculosis
Answer: b. Staphylococcus aureus

669. The standard treatment for anaphylaxis includes:

a. Antihistamines
b. Corticosteroids
c. Epinephrine
d. Oxygen therapy
Answer: c. Epinephrine

670. The primary screening test for colorectal cancer is:

a. Colonoscopy
b. Fecal occult blood test
c. Sigmoidoscopy
d. CT colonography
Answer: b. Fecal occult blood test

671. The first-line treatment for type 2 diabetes is: \

a. Metformin
b. Insulin
c. Sulfonylureas
d. DPP-4 inhibitors
Answer: a. Metformin

672. The classic triad of symptoms in nephrotic syndrome includes:

a. Edema, proteinuria, and hyperlipidemia


b. Hematuria, proteinuria, and hypertension
c. Edema, hematuria, and hypertension
d. Hypertension, fatigue, and proteinuria
Answer: a. Edema, proteinuria, and hyperlipidemia
673. The most common cause of secondary amenorrhea is:

a. Pregnancy
b. Polycystic ovary syndrome
c. Thyroid disorders
d. Stress
Answer: a. Pregnancy

674. The primary characteristic of nephrotic syndrome is:

a. Hematuria
b. Proteinuria
c. Hypertension
d. Oliguria
Answer: b. Proteinuria

675. The first-line treatment for hyperlipidemia is:

a. Statins
b. Fibrates
c. Niacin
d. Bile acid sequestrants
Answer: a. Statins

676. The classic symptom of pulmonary embolism is:

a. Chest pain
b. Hemoptysis
c. Shortness of breath
d. Cough
Answer: c. Shortness of breath

677. The most common site for ectopic pregnancy is:

a. Ovary
b. Uterine cornua
c. Abdominal cavity
d. Fallopian tube
Answer: d. Fallopian tube

678. The standard treatment for hypertension in a patient with heart failure
is:

a. Calcium channel blockers


b. Diuretics
c. Beta-blockers
d. ACE inhibitors
Answer: d. ACE inhibitors

679. The best initial test for diagnosing diabetes mellitus is:

a. Fasting blood glucose


b. Oral glucose tolerance test
c. HbA1c
d. Random blood glucose
Answer: a. Fasting blood glucose

680. The primary cause of iron deficiency anemia is:

a. Chronic disease
b. Blood loss
c. Increased demand
d. Malabsorption
Answer: b. Blood loss

681. The classic finding in a patient with Cushing's syndrome is:

a. Moon facies
b. Weight loss
c. Hyperpigmentation
d. Dehydration
Answer: a. Moon facies

682. The most common cause of hyperkalemia is:


a. Renal failure
b. Adrenal insufficiency
c. Excessive potassium intake
d. Medications
Answer: a. Renal failure

683. The first-line treatment for osteoporosis is:

a. Calcium and vitamin D supplementation


b. Bisphosphonates
c. Estrogen replacement therapy
d. Hormone replacement therapy
Answer: b. Bisphosphonates

684. The most common cause of hypothyroidism is:

a. Hashimoto's thyroiditis
b. Iodine deficiency
c. Thyroidectomy
d. Pituitary disease
Answer: a. Hashimoto's thyroiditis

685. The classic sign of hyperventilation is:

a. Bradypnea
b. Tachypnea
c. Apnea
d. Dyspnea
Answer: b. Tachypnea

686. The primary risk factor for stroke is:

a. Diabetes
b. Smoking
c. Hypertension
d. Hyperlipidemia
Answer: c. Hypertension
687. The most common cause of acute renal failure is:

a. Pre-renal azotemia
b. Post-renal azotemia
c. Intrinsic renal failure
d. Acute tubular necrosis
Answer: a. Pre-renal azotemia

688. The first-line treatment for allergic rhinitis is:

a. Oral antihistamines
b. Nasal corticosteroids
c. Decongestants
d. Leukotriene receptor antagonists
Answer: b. Nasal corticosteroids

689. The most common symptom of left ventricular heart failure is:

a. Peripheral edema
b. Dyspnea on exertion
c. Chest pain
d. Fatigue
Answer: b. Dyspnea on exertion

690. The standard initial management for acute coronary syndrome is:

a. Anticoagulants
b. Aspirin and clopidogrel
c. Thrombolytics
d. Beta-blockers
Answer: b. Aspirin and clopidogrel

691. The classic triad of symptoms in meningitis includes:

a. Fever, headache, and neck stiffness


b. Fever, cough, and dyspnea
c. Headache, nausea, and vomiting
d. Fever, rash, and arthralgia
Answer: a. Fever, headache, and neck stiffness

692. The most common cause of hyperthyroidism is:

a. Hashimoto's thyroiditis
b. Graves' disease
c. Thyroid adenoma
d. Pituitary adenoma
Answer: b. Graves' disease

693. The primary treatment for an acute asthma attack is:

a. Inhaled corticosteroids
b. Long-acting beta-agonists
c. Short-acting beta-agonists
d. Leukotriene receptor antagonists
Answer: c. Short-acting beta-agonists

694. The most common complication of diabetes mellitus is:

a. Neuropathy
b. Retinopathy
c. Nephropathy
d. Cardiovascular disease
Answer: d. Cardiovascular disease

695. The first-line medication for hypertension in pregnancy is:

a. Methyldopa
b. Labetalol
c. Nifedipine
d. ACE inhibitors
Answer: a. Methyldopa

696. The classic symptom of peptic ulcer disease is:


a. Abdominal pain
b. Diarrhea
c. Weight loss
d. Nausea
Answer: a. Abdominal pain

697. The most common type of anemia in the general population is:

a. Iron deficiency anemia


b. B12 deficiency anemia
c. Aplastic anemia
d. Hemolytic anemia
Answer: a. Iron deficiency anemia

698. The best initial test for diagnosing heart failure is:

a. Echocardiogram
b. Chest X-ray
c. BNP levels
d. ECG
Answer: c. BNP levels

699. The leading cause of preventable death worldwide is:

a. Alcohol use
b. Smoking
c. Obesity
d. Inactivity
Answer: b. Smoking

700. The most common cause of lower gastrointestinal bleeding is:

a. Hemorrhoids
b. Diverticulosis
c. Colorectal cancer
d. Inflammatory bowel disease
Answer: b. Diverticulosis
701. The classic sign of congestive heart failure is:

a. JVD (jugular venous distension)


b. S3 heart sound
c. Peripheral edema
d. All of the above
Answer: d. All of the above

702. The first-line treatment for major depressive disorder is:

a. Psychotherapy
b. SSRIs
c. Tricyclic antidepressants
d. Mood stabilizers
Answer: b. SSRIs

703. The most sensitive test for detecting liver disease is:

a. ALT
b. AST
c. ALP
d. Bilirubin
Answer: a. ALT

704. The primary treatment for anaphylaxis is:

a. Antihistamines
b. Epinephrine
c. Corticosteroids
d. Saline infusion
Answer: b. Epinephrine

705. The leading cause of chronic kidney disease is:

a. Diabetes mellitus
b. Hypertension
c. Glomerulonephritis
d. Polycystic kidney disease
Answer: a. Diabetes mellitus

706. The classic symptom of myocardial infarction in women may be:

a. Chest pain
b. Jaw pain
c. Back pain
d. All of the above
Answer: d. All of the above

707. The most common cause of community-acquired pneumonia is:

a. Streptococcus pneumoniae
b. Haemophilus influenzae
c. Mycoplasma pneumoniae
d. Legionella pneumophila
Answer: a. Streptococcus pneumoniae

708. The best initial treatment for hypertension in a diabetic patient is:

a. Calcium channel blockers


b. ACE inhibitors
c. Diuretics
d. Beta-blockers
Answer: b. ACE inhibitors

709. The classic finding in a patient with acute pancreatitis is:

a. Elevated blood glucose


b. Elevated amylase and lipase
c. Hypocalcemia
d. Jaundice
Answer: b. Elevated amylase and lipase

710. The most effective strategy for preventing cardiovascular disease is:
a. Weight loss
b. Smoking cessation
c. Exercise
d. Low-sodium diet
Answer: b. Smoking cessation

711. The primary test for diagnosing diabetes mellitus is:

a. Fasting plasma glucose


b. HbA1c
c. Oral glucose tolerance test
d. Random plasma glucose
Answer: a. Fasting plasma glucose

712. The most common type of cancer in women is:

a. Breast cancer
b. Lung cancer
c. Colorectal cancer
d. Uterine cancer
Answer: a. Breast cancer

713. The primary method of diagnosing coronary artery disease is:

a. Stress test
b. Coronary angiography
c. Cardiac MRI
d. Echocardiogram
Answer: b. Coronary angiography

714. The best initial therapy for a patient with hyperkalemia is:

a. Calcium gluconate
b. Insulin and glucose
c. Sodium bicarbonate
d. Diuretics
Answer: b. Insulin and glucose
715. The most common cause of acute bronchitis is:

a. Bacterial infection
b. Viral infection
c. Allergies
d. Environmental irritants
Answer: b. Viral infection

716. The standard treatment for osteoporosis is:

a. Calcium and vitamin D


b. Bisphosphonates
c. Hormone replacement therapy
d. Estrogen
Answer: b. Bisphosphonates

717. The classic symptom of diabetic ketoacidosis is:

a. Polyuria
b. Polydipsia
c. Abdominal pain
d. All of the above
Answer: d. All of the above

718. The most common cause of secondary hypertension is:

a. Cushing's syndrome
b. Renal artery stenosis
c. Hyperaldosteronism
d. Obstructive sleep apnea
Answer: d. Obstructive sleep apnea

719. The first-line treatment for gout is:

a. Allopurinol
b. Colchicine
c. NSAIDs
d. Corticosteroids
Answer: c. NSAIDs

720. The most common complication of cirrhosis is:

a. Hepatic encephalopathy
b. Ascites
c. Esophageal varices
d. Hepatorenal syndrome
Answer: b. Ascites

721. The most common type of skin cancer is:

a. Melanoma
b. Basal cell carcinoma
c. Squamous cell carcinoma
d. Merkel cell carcinoma
Answer: b. Basal cell carcinoma

722. The classic triad of symptoms in nephrotic syndrome includes:

a. Edema, proteinuria, and hyperlipidemia


b. Hematuria, flank pain, and fever
c. Edema, hematuria, and hypertension
d. Dysuria, urgency, and frequency
Answer: a. Edema, proteinuria, and hyperlipidemia

723. The drug of choice for hyperlipidemia is:

a. Statins
b. Fibrates
c. Niacin
d. Omega-3 fatty acids
Answer: a. Statins

724. The most common cause of acute renal failure is:


a. Prerenal azotemia
b. Intrinsic renal failure
c. Postrenal obstruction
d. Acute tubular necrosis
Answer: d. Acute tubular necrosis

725. The primary cause of Addison's disease is:

a. Autoimmune destruction of the adrenal glands


b. Tumor of the adrenal cortex
c. Pituitary gland dysfunction
d. Congenital adrenal hyperplasia
Answer: a. Autoimmune destruction of the adrenal glands

726. The first-line treatment for obsessive-compulsive disorder is:

a. SSRIs
b. CBT (Cognitive Behavioral Therapy)
c. Tricyclic antidepressants
d. Benzodiazepines
Answer: a. SSRIs

727. The classic finding in a patient with hypothyroidism is:

a. Weight loss
b. Bradycardia
c. Hyperactivity
d. Warm, moist skin
Answer: b. Bradycardia

728. The best initial management for a patient with anaphylaxis is:

a. Antihistamines
b. Intravenous fluids
c. Epinephrine
d. Corticosteroids
Answer: c. Epinephrine
729. The most sensitive imaging test for detecting a pulmonary embolism is:

a. Chest X-ray
b. CT pulmonary angiography
c. V/Q scan
d. Doppler ultrasound
Answer: b. CT pulmonary angiography

730. The most common cause of chronic cough in adults is:

a. Asthma
b. Chronic bronchitis
c. GERD (Gastroesophageal reflux disease)
d. Postnasal drip
Answer: d. Postnasal drip

731. The preferred first-line therapy for hypertension in the elderly is:

a. Calcium channel blockers


b. ACE inhibitors
c. Diuretics
d. Beta-blockers
Answer: c. Diuretics

732. The most common pathogen causing urinary tract infections in women
is:

a. Escherichia coli
b. Staphylococcus saprophyticus
c. Klebsiella pneumoniae
d. Enterococcus faecalis
Answer: a. Escherichia coli

733. The drug of choice for hypertension in pregnancy is:

a. Labetalol
b. Hydrochlorothiazide
c. Amlodipine
d. Enalapril
Answer: a. Labetalol

734. The classic presentation of appendicitis includes:

a. Right lower quadrant pain, nausea, and vomiting


b. Left lower quadrant pain, diarrhea, and fever
c. Epigastric pain radiating to the back
d. Severe chest pain and shortness of breath
Answer: a. Right lower quadrant pain, nausea, and vomiting

735. The most common electrolyte imbalance in hospitalized patients is:

a. Hyperkalemia
b. Hyponatremia
c. Hypercalcemia
d. Hypomagnesemia
Answer: b. Hyponatremia

736. The most effective first-line treatment for severe asthma exacerbation
is:

a. Inhaled corticosteroids
b. Systemic corticosteroids
c. Short-acting beta-agonists
d. Long-acting beta-agonists
Answer: c. Short-acting beta-agonists

737. The classic sign of aortic stenosis is:

a. Pulsus paradoxus
b. Systolic ejection murmur
c. Diastolic murmur
d. Continuous murmur
Answer: b. Systolic ejection murmur
738. The most common cause of hemoptysis is:

a. Lung cancer
b. Tuberculosis
c. Bronchitis
d. Pulmonary embolism
Answer: c. Bronchitis

739. The first-line treatment for primary hyperaldosteronism is:

a. Spironolactone
b. Eplerenone
c. Amiloride
d. ACE inhibitors
Answer: a. Spironolactone

740. The most common cause of septic arthritis in adults is:

a. Staphylococcus aureus
b. Streptococcus pneumoniae
c. Neisseria gonorrhoeae
d. Escherichia coli
Answer: a. Staphylococcus aureus

741. The classic symptom of congestive heart failure is:

a. Paroxysmal nocturnal dyspnea


b. Syncope
c. Orthostatic hypotension
d. Palpitations
Answer: a. Paroxysmal nocturnal dyspnea

742. The most common type of intracranial hemorrhage is:

a. Subdural hematoma
b. Epidural hematoma
c. Intracerebral hemorrhage
d. Subarachnoid hemorrhage
Answer: c. Intracerebral hemorrhage

743. The classic sign of multiple sclerosis is:

a. Ataxia
b. Hemiparesis
c. Diplopia
d. Paresthesia
Answer: c. Diplopia

744. The initial treatment for a patient with a suspected pulmonary embolism
is:

a. Warfarin
b. Heparin
c. Aspirin
d. Clopidogrel
Answer: b. Heparin

745. The first-line treatment for schizophrenia is:

a. SSRIs
b. Atypical antipsychotics
c. Benzodiazepines
d. Mood stabilizers
Answer: b. Atypical antipsychotics

746. The most common cause of infectious diarrhea in adults is:

a. Norovirus
b. Salmonella
c. Shigella
d. Campylobacter
Answer: a. Norovirus

747. The standard initial test for suspected celiac disease is:
a. Anti-tissue transglutaminase antibodies
b. Endoscopy with biopsy
c. Stool analysis
d. Serum electrolytes
Answer: a. Anti-tissue transglutaminase antibodies

748. The most common site for a peptic ulcer is:

a. Duodenum
b. Stomach
c. Esophagus
d. Jejunum
Answer: a. Duodenum

749. The classic presentation of hyperparathyroidism includes:

a. Stones, bones, groans, and psychiatric overtones


b. Weakness, weight gain, and cold intolerance
c. Numbness, tingling, and cramps
d. Abdominal pain and jaundice
Answer: a. Stones, bones, groans, and psychiatric overtones

750. The most common cause of acute bacterial sinusitis is:

a. Streptococcus pneumoniae
b. Haemophilus influenzae
c. Moraxella catarrhalis
d. Staphylococcus aureus
Answer: a. Streptococcus pneumoniae

751. The most common cause of chronic liver disease in the world is:

a. Alcoholic liver disease


b. Non-alcoholic fatty liver disease
c. Viral hepatitis
d. Autoimmune hepatitis
Answer: c. Viral hepatitis
752. The primary treatment for hyperglycemia in diabetic ketoacidosis is:

a. Intravenous fluids
b. Insulin therapy
c. Electrolyte replacement
d. Oral hypoglycemics
Answer: b. Insulin therapy

753. The classic triad of symptoms in Wernicke's encephalopathy includes:

a. Ataxia, confusion, ophthalmoplegia


b. Nystagmus, memory loss, hemiparesis
c. Dementia, seizures, tremors
d. Dysphasia, agnosia, apraxia
Answer: a. Ataxia, confusion, ophthalmoplegia

754. The most common cardiac arrhythmia in patients with heart failure is:

a. Atrial fibrillation
b. Ventricular tachycardia
c. Sinus bradycardia
d. Premature ventricular contractions
Answer: a. Atrial fibrillation

755. The best initial test for suspected coronary artery disease is:

a. Stress echocardiography
b. Cardiac catheterization
c. Coronary CT angiography
d. Standard ECG
Answer: a. Stress echocardiography

756. The most common cause of pancreatitis is:

a. Alcohol consumption
b. Gallstones
c. Hyperlipidemia
d. Medications
Answer: b. Gallstones

757. The classic presentation of diabetic retinopathy includes:

a. Cotton wool spots and microaneurysms


b. Flame-shaped hemorrhages and exudates
c. Vision loss and photophobia
d. Scotomas and increased intraocular pressure
Answer: a. Cotton wool spots and microaneurysms

758. The most common cause of peptic ulcer disease is:

a. H. pylori infection
b. NSAID use
c. Stress
d. Alcohol consumption
Answer: a. H. pylori infection

759. The classic symptoms of aortic dissection include:

a. Sudden chest pain and back pain


b. Chest pain radiating to the left arm
c. Dizziness and syncope
d. Severe headache and neck stiffness
Answer: a. Sudden chest pain and back pain

760. The best initial treatment for an asthma attack is:

a. Oral corticosteroids
b. Short-acting beta-agonists
c. Long-acting beta-agonists
d. Inhaled corticosteroids
Answer: b. Short-acting beta-agonists

761. The classic symptom of hyperthyroidism is:


a. Weight gain
b. Fatigue
c. Heat intolerance
d. Cold intolerance
Answer: c. Heat intolerance

762. The most common cause of secondary hypertension is:

a. Renal artery stenosis


b. Cushing's syndrome
c. Hyperaldosteronism
d. Sleep apnea
Answer: a. Renal artery stenosis

763. The primary goal in managing chronic obstructive pulmonary disease


(COPD) is:

a. Reduce inflammation
b. Improve airflow
c. Increase oxygen saturation
d. Decrease mucus production
Answer: b. Improve airflow

764. The first-line treatment for gout is:

a. Allopurinol
b. Colchicine
c. Indomethacin
d. Corticosteroids
Answer: b. Colchicine

765. The classic sign of pulmonary embolism is:

a. Tachycardia
b. Hemoptysis
c. Chest pain
d. All of the above
Answer: d. All of the above

766. The best first-line treatment for major depressive disorder is:

a. CBT
b. SSRIs
c. Tricyclic antidepressants
d. MAO inhibitors
Answer: b. SSRIs

767. The primary cause of secondary amenorrhea is:

a. Pregnancy
b. Stress
c. Hormonal imbalance
d. Menopause
Answer: a. Pregnancy

768. The classic sign of diabetic neuropathy is:

a. Symmetric distal paresthesia


b. Radicular pain
c. Unilateral weakness
d. Numbness in the face
Answer: a. Symmetric distal paresthesia

769. The most common pathogen in community-acquired pneumonia is:

a. Streptococcus pneumoniae
b. Haemophilus influenzae
c. Mycoplasma pneumoniae
d. Chlamydia pneumoniae
Answer: a. Streptococcus pneumoniae

770. The classic presentation of cholecystitis includes:


a. Right upper quadrant pain, fever, and Murphy's sign
b. Left upper quadrant pain, jaundice, and fever
c. Epigastric pain radiating to the back
d. Abdominal distension and vomiting
Answer: a. Right upper quadrant pain, fever, and Murphy's sign

771. The most common cause of hemolytic anemia is:

a. Autoimmune hemolytic anemia


b. Sickle cell disease
c. Thalassemia
d. Hemolytic uremic syndrome
Answer: a. Autoimmune hemolytic anemia

772. The classic symptom of systemic lupus erythematosus (SLE) is:

a. Butterfly rash
b. Joint pain
c. Photosensitivity
d. All of the above
Answer: d. All of the above

773. The most common cause of metabolic alkalosis is:

a. Vomiting
b. Diarrhea
c. Renal failure
d. Hyperventilation
Answer: a. Vomiting

774. The best initial treatment for a patient with opioid overdose is:

a. Flumazenil
b. Naloxone
c. Activated charcoal
d. Supportive care
Answer: b. Naloxone
775. The classic sign of Bell's palsy is:

a. Unilateral facial droop


b. Difficulty swallowing
c. Hearing loss
d. Nystagmus
Answer: a. Unilateral facial droop

776. The most common type of stroke is:

a. Ischemic stroke
b. Hemorrhagic stroke
c. Transient ischemic attack
d. Lacunar stroke
Answer: a. Ischemic stroke

777. The primary treatment for acute coronary syndrome is:

a. Aspirin
b. Clopidogrel
c. Beta-blockers
d. Anticoagulation
Answer: a. Aspirin

778. The most common cause of obstructive sleep apnea is:

a. Obesity
b. Alcohol use
c. Smoking
d. Sedative medications
Answer: a. Obesity

779. The classic sign of cervical radiculopathy is:

a. Weakness in the arm


b. Numbness in the leg
c. Back pain
d. Shoulder pain
Answer: a. Weakness in the arm

780. The best initial test for suspected deep vein thrombosis is:

a. Doppler ultrasound
b. CT angiography
c. MRI
d. D-dimer test
Answer: a. Doppler ultrasound

781. The most common cause of hypercalcemia is:

a. Vitamin D toxicity
b. Primary hyperparathyroidism
c. Malignancy
d. Renal failure
Answer: b. Primary hyperparathyroidism

782. The classic presentation of acute appendicitis includes:

a. Right upper quadrant pain


b. Periumbilical pain that migrates to the right lower quadrant
c. Fever and diarrhea
d. Bloating and flatulence
Answer: b. Periumbilical pain that migrates to the right lower quadrant

783. The primary treatment for hypertension in a patient with heart failure is:

a. Beta-blockers
b. ACE inhibitors
c. Diuretics
d. Calcium channel blockers
Answer: b. ACE inhibitors

784. The most common cause of iron deficiency anemia in adults is:
a. Dietary deficiency
b. Blood loss
c. Malabsorption
d. Chronic disease
Answer: b. Blood loss

785. The first-line treatment for hypertension in a pregnant woman is:

a. ACE inhibitors
b. Calcium channel blockers
c. Methyldopa
d. Thiazide diuretics
Answer: c. Methyldopa

786. The classic triad of symptoms in nephrotic syndrome includes:

a. Edema, proteinuria, and hypoalbuminemia


b. Hematuria, hypertension, and flank pain
c. Polyuria, polydipsia, and weight loss
d. Abdominal pain, jaundice, and dark urine
Answer: a. Edema, proteinuria, and hypoalbuminemia

787. The most common organism causing urinary tract infections in women
is:

a. E. coli
b. Klebsiella
c. Enterococcus
d. Staphylococcus saprophyticus
Answer: a. E. coli

788. The classic symptom of Hashimoto's thyroiditis is:

a. Weight gain
b. Cold intolerance
c. Dry skin
d. All of the above
Answer: d. All of the above

789. The first-line treatment for chronic obstructive pulmonary disease


(COPD) is:

a. Long-acting beta-agonists
b. Short-acting beta-agonists
c. Inhaled corticosteroids
d. Oxygen therapy
Answer: a. Long-acting beta-agonists

790. The classic presentation of a myocardial infarction includes:

a. Sudden chest pain radiating to the left arm


b. Pain relieved by rest
c. Sharp pain with breathing
d. Palpitations and dizziness
Answer: a. Sudden chest pain radiating to the left arm

791. The most common complication of diabetes mellitus is:

a. Cardiovascular disease
b. Peripheral neuropathy
c. Nephropathy
d. Retinopathy
Answer: a. Cardiovascular disease

792. The primary test for diagnosing heart failure is:

a. Chest X-ray
b. ECG
c. Echocardiogram
d. BNP levels
Answer: c. Echocardiogram

793. The classic symptom of a tension pneumothorax is:


a. Hypotension
b. Sudden onset chest pain
c. Diminished breath sounds
d. Tracheal deviation
Answer: d. Tracheal deviation

794. The first-line treatment for anaphylaxis is:

a. Corticosteroids
b. Antihistamines
c. Epinephrine
d. Oxygen
Answer: c. Epinephrine

795. The most common cause of aortic stenosis in older adults is:

a. Congenital bicuspid aortic valve


b. Rheumatic fever
c. Degenerative calcific changes
d. Infective endocarditis
Answer: c. Degenerative calcific changes

796. The classic sign of rheumatoid arthritis is:

a. Symmetric joint involvement


b. Asymmetric joint involvement
c. Morning stiffness lasting less than 30 minutes
d. Fever and rash
Answer: a. Symmetric joint involvement

797. The primary mechanism of action of statins is:

a. Inhibiting cholesterol absorption


b. Inhibiting HMG-CoA reductase
c. Promoting bile acid excretion
d. Increasing HDL levels
Answer: b. Inhibiting HMG-CoA reductase
798. The classic presentation of a stroke includes:

a. Sudden weakness or numbness on one side of the body


b. Severe headache with nausea
c. Visual disturbances
d. All of the above
Answer: d. All of the above

799. The most common cause of hyperthyroidism is

a. Hashimoto's thyroiditis
b. Toxic multinodular goiter
c. Graves' disease
d. Thyroid carcinoma
Answer: c. Graves' disease

800. The best initial diagnostic test for suspected acute pancreatitis is:

a. CT scan
b. MRI
c. Serum lipase
d. Abdominal ultrasound
Answer: c. Serum lipase

801. The classic triad of symptoms in Cushing's syndrome includes:

a. Moon facies, hypertension, and striae


b. Weight loss, fever, and fatigue
c. Fatigue, weakness, and hypoglycemia
d. Hyperpigmentation, hypotension, and weight loss
Answer: a. Moon facies, hypertension, and striae

802. The first-line treatment for hyperlipidemia is:

a. Lifestyle changes
b. Statins
c. Fibrates
d. Niacin
Answer: a. Lifestyle changes

803. The most common cause of chronic kidney disease is:

a. Diabetes mellitus
b. Hypertension
c. Glomerulonephritis
d. Polycystic kidney disease
Answer: a. Diabetes mellitus

804. The classic symptom of bacterial meningitis is:

a. Nuchal rigidity
b. Fever and rash
c. Headache and photophobia
d. All of the above
Answer: d. All of the above

805. The most common cause of a viral sore throat is:

a. Streptococcus
b. Epstein-Barr virus
c. Adenovirus
d. Cytomegalovirus
Answer: c. Adenovirus

806. The classic symptom of chronic venous insufficiency is:

a. Edema
b. Claudication
c. Cold extremities
d. Cyanosis
Answer: a. Edema

807. The first-line treatment for acute migraine is:


a. Triptans
b. NSAIDs
c. Ergotamines
d. Antidepressants
Answer: a. Triptans

808. The most common site of a peptic ulcer is:

a. Duodenum
b. Stomach
c. Esophagus
d. Jejunum
Answer: a. Duodenum

809. The classic symptom of a subdural hematoma is:

a. Rapid onset of headache


b. Gradual deterioration of consciousness
c. Seizures
d. Focal neurological deficits
Answer: b. Gradual deterioration of consciousness

810. The most common cause of acute liver failure is:

a. Viral hepatitis
b. Alcoholic liver disease
c. Drug-induced liver injury
d. Autoimmune hepatitis
Answer: c. Drug-induced liver injury

811. The primary treatment for hyperkalemia is:

a. Calcium gluconate
b. Insulin and glucose
c. Sodium bicarbonate
d. Diuretics
Answer: b. Insulin and glucose
812. The classic symptom of aortic regurgitation is:

a. Syncope
b. Palpitations
c. Diastolic murmur
d. Systolic ejection click
Answer: c. Diastolic murmur

813. The most common cause of community-acquired pneumonia is:

a. Staphylococcus aureus
b. Streptococcus pneumoniae
c. Mycoplasma pneumoniae
d. Haemophilus influenzae
Answer: b. Streptococcus pneumoniae

814. The classic feature of osteoarthritis is:

a. Morning stiffness lasting more than 30 minutes


b. Swelling of the joints
c. Joint pain worsened with activity
d. Fever and malaise
Answer: c. Joint pain worsened with activity

815. The most common cause of peptic ulcer disease is:

a. NSAID use
b. Helicobacter pylori infection
c. Stress
d. Alcohol consumption
Answer: b. Helicobacter pylori infection

816. The primary test for diagnosing diabetes mellitus is:

a. Fasting blood glucose


b. Random blood glucose
c. HbA1c
d. Oral glucose tolerance test
Answer: c. HbA1c

817. The classic triad of symptoms in nephritic syndrome includes:

a. Edema, hypertension, and hematuria


b. Proteinuria, hyperlipidemia, and edema
c. Edema, hypercalcemia, and weight loss
d. Fatigue, pallor, and jaundice
Answer: a. Edema, hypertension, and hematuria

818. The most common cause of bacterial endocarditis in IV drug users is:

a. Streptococcus viridans
b. Staphylococcus aureus
c. Enterococcus
d. HACEK organisms
Answer: b. Staphylococcus aureus

819. The primary mechanism of action of aspirin is:

a. COX inhibition
b. Thromboxane synthesis
c. Platelet aggregation
d. Anticoagulation
Answer: a. COX inhibition

820. The most common cause of hypothyroidism in the developed world is:

a. Hashimoto's thyroiditis
b. Iodine deficiency
c. Surgical removal of the thyroid
d. Thyroiditis
Answer: a. Hashimoto's thyroiditis

821. The classic symptom of a tension pneumothorax is:


a. Decreased breath sounds
b. Tracheal deviation away from the affected side
c. Hypotension
d. Cyanosis
Answer: b. Tracheal deviation away from the affected side

822. The first-line treatment for acute gout is:

a. Corticosteroids
b. Allopurinol
c. Colchicine
d. Non-steroidal anti-inflammatory drugs (NSAIDs)
Answer: d. Non-steroidal anti-inflammatory drugs (NSAIDs)

823. The classic feature of chronic obstructive pulmonary disease (COPD) is:

a. Restrictive lung disease


b. Airflow limitation that is not fully reversible
c. Increased lung volumes
d. Decreased respiratory rate
Answer: b. Airflow limitation that is not fully reversible

824. The most common cause of acute pancreatitis is:

a. Alcohol abuse
b. Gallstones
c. Hyperlipidemia
d. Medications
Answer: b. Gallstones

825. The primary hormone responsible for the regulation of calcium levels in
the blood is:

a. Parathyroid hormone
b. Calcitonin
c. Vitamin D
d. Aldosterone
Answer: a. Parathyroid hormone

826. The classic symptom of appendicitis is:

a. Right upper quadrant pain


b. Flank pain
c. Right lower quadrant pain
d. Severe abdominal cramps
Answer: c. Right lower quadrant pain

827. The most common cause of acute liver injury is:

a. Viral hepatitis
b. Alcoholic liver disease
c. Drug-induced liver injury
d. Non-alcoholic fatty liver disease
Answer: c. Drug-induced liver injury

828. The primary mechanism of action of beta-blockers is:

a. Blocking calcium channels


b. Inhibiting renin release
c. Decreasing heart rate and contractility
d. Vasodilation
Answer: c. Decreasing heart rate and contractility

829. The classic symptom of hyperthyroidism is:

a. Weight gain
b. Cold intolerance
c. Weight loss
d. Bradycardia
Answer: c. Weight loss

830. The most common presenting symptom of pulmonary embolism is:


a. Chest pain
b. Cough
c. Shortness of breath
d. Hemoptysis
Answer: c. Shortness of breath

831. The most common type of stroke is:

a. Hemorrhagic
b. Ischemic
c. Transient ischemic attack (TIA)
d. Lacunar
Answer: b. Ischemic

832. The primary cause of coronary artery disease is:

a. Hypertension
b. Hyperlipidemia
c. Smoking
d. Diabetes mellitus
Answer: b. Hyperlipidemia

833. The classic sign of liver cirrhosis is:

a. Jaundice
b. Palmar erythema
c. Ascites
d. Spider angiomata
Answer: c. Ascites

834. The most common congenital heart defect is:

a. Ventricular septal defect


b. Atrial septal defect
c. Tetralogy of Fallot
d. Coarctation of the aorta
Answer: a. Ventricular septal defect
835. The primary treatment for anaphylaxis is:

a. Antihistamines
b. Corticosteroids
c. Epinephrine
d. Oxygen
Answer: c. Epinephrine

836. The most common cause of secondary hypertension is:

a. Kidney disease
b. Endocrine disorders
c. Sleep apnea
d. Obesity
Answer: a. Kidney disease

837. The primary function of the kidneys is:

a. Hormone production
b. Regulation of blood pressure
c. Waste excretion and fluid balance
d. Electrolyte regulation
Answer: c. Waste excretion and fluid balance

838. The classic triad of symptoms in renal cell carcinoma includes:

a. Hematuria, flank pain, and weight loss


b. Hypertension, hematuria, and fever
c. Anemia, pain, and jaundice
d. Back pain, fever, and cough
Answer: a. Hematuria, flank pain, and weight loss

839. The most common cause of hypothyroidism in developing countries is:

a. Hashimoto's thyroiditis
b. Iodine deficiency
c. Surgical removal of the thyroid
d. Thyroiditis
Answer: b. Iodine deficiency

840. The primary treatment for hyperlipidemia is:

a. Statins
b. Fibrates
c. Niacin
d. Diet modification
Answer: a. Statins

841. The classic symptom of heart failure is:

a. Chest pain
b. Fatigue and dyspnea on exertion
c. Palpitations
d. Syncope
Answer: b. Fatigue and dyspnea on exertion

842. The most common cause of bacterial meningitis in adults is:

a. Streptococcus pneumoniae
b. Neisseria meningitidis
c. Listeria monocytogenes
d. Haemophilus influenzae
Answer: a. Streptococcus pneumoniae

843. The primary mechanism of action of ACE inhibitors is:

a. Vasodilation
b. Inhibition of angiotensin II formation
c. Decreasing blood volume
d. Blocking calcium channels
Answer: b. Inhibition of angiotensin II formation

844. The classic sign of iron deficiency anemia is:


a. Microcytic red blood cells
b. Macrocytic red blood cells
c. Normocytic red blood cells
d. Spherocytes
Answer: a. Microcytic red blood cells

845. The most common initial symptom of diabetes mellitus is:

a. Polyuria
b. Polydipsia
c. Polyphagia
d. Weight loss
Answer: a. Polyuria

846. The first-line treatment for asthma exacerbation is:

a. Inhaled corticosteroids
b. Short-acting beta-agonists
c. Long-acting beta-agonists
d. Leukotriene receptor antagonists
Answer: b. Short-acting beta-agonists

847. The most common site for atherosclerotic plaque formation is:

a. Aorta
b. Coronary arteries
c. Carotid arteries
d. Peripheral arteries
Answer: a. Aorta

848. The primary complication of untreated hypertension is:

a. Stroke
b. Myocardial infarction
c. Renal failure
d. Heart failure
Answer: a. Stroke
849. The primary diagnostic test for pulmonary embolism is:

a. Chest X-ray
b. CT pulmonary angiography
c. MRI
d. Ventilation-perfusion scan
Answer: b. CT pulmonary angiography

850. The classic symptom of diabetic ketoacidosis is:

a. Kussmaul respirations
b. Polyuria
c. Nausea and vomiting
d. Abdominal pain
Answer: a. Kussmaul respirations

851. The most common presenting symptom of esophageal cancer is:

a. Dysphagia
b. Weight loss
c. Chest pain
d. Hematemesis
Answer: a. Dysphagia

852. The primary treatment for major depressive disorder is:

a. Psychotherapy
b. Antidepressants
c. Electroconvulsive therapy
d. Lifestyle changes
Answer: b. Antidepressants

853. The classic symptom of a myocardial infarction is:

a. Chest pressure or pain


b. Shortness of breath
c. Nausea
d. Sweating
Answer: a. Chest pressure or pain

854. The most common type of skin cancer is:

a. Melanoma
b. Squamous cell carcinoma
c. Basal cell carcinoma
d. Merkel cell carcinoma
Answer: c. Basal cell carcinoma

855. The primary treatment for osteoporosis is:

a. Bisphosphonates
b. Calcium and vitamin D supplementation
c. Hormone replacement therapy
d. Weight-bearing exercise
Answer: a. Bisphosphonates

856. The classic sign of meningitis is:

a. Kernig's sign
b. Brudzinski's sign
c. Photophobia
d. All of the above
Answer: d. All of the above

857. The primary cause of chronic kidney disease is:

a. Diabetes mellitus
b. Hypertension
c. Glomerulonephritis
d. Polycystic kidney disease
Answer: a. Diabetes mellitus

858. The most common infectious cause of diarrhea in children is:


a. Rotavirus
b. Norovirus
c. Escherichia coli
d. Salmonella
Answer: a. Rotavirus

859. The classic feature of Cushing's syndrome is:

a. Weight loss
b. Moon facies
c. Hyperkalemia
d. Hyponatremia
Answer: b. Moon facies

860. The primary treatment for chronic migraines is:

a. Analgesics
b. Abortive medications
c. Preventive medications
d. Lifestyle changes
Answer: c. Preventive medications

861. The most common cause of acute pancreatitis is:

a. Alcohol consumption
b. Gallstones
c. Viral infection
d. Hyperlipidemia
Answer: b. Gallstones

862. The primary characteristic of nephrotic syndrome is:

a. Hematuria
b. Proteinuria
c. Hypertension
d. Edema
Answer: b. Proteinuria
863. The first-line treatment for hypertension in pregnancy is:

a. Methyldopa
b. Lisinopril
c. Amlodipine
d. Hydrochlorothiazide
Answer: a. Methyldopa

864. The most common organism causing urinary tract infections is:

a. Escherichia coli
b. Klebsiella pneumoniae
c. Staphylococcus saprophyticus
d. Proteus mirabilis
Answer: a. Escherichia coli

865. The classic sign of hyperthyroidism is:

a. Weight gain
b. Bradycardia
c. Exophthalmos
d. Cold intolerance
Answer: c. Exophthalmos

866. The best initial test for diagnosing heart failure is:

a. Chest X-ray
b. EKG
c. B-type natriuretic peptide (BNP)
d. Echocardiogram
Answer: c. B-type natriuretic peptide (BNP)

867. The primary mechanism of action of beta-blockers is:

a. Calcium channel blockade


b. Inhibition of renin release
c. Decreasing heart rate and contractility
d. Vasodilation
Answer: c. Decreasing heart rate and contractility

868. The most common cause of acute gastroenteritis is:

a. Bacterial infection
b. Viral infection
c. Parasitic infection
d. Food allergies
Answer: b. Viral infection

869. The characteristic feature of rheumatoid arthritis is:

a. Asymmetrical joint involvement


b. Morning stiffness
c. Osteophyte formation
d. Joint effusion
Answer: b. Morning stiffness

870. The first-line treatment for generalized anxiety disorder is:

a. Benzodiazepines
b. SSRIs
c. SNRIs
d. Buspirone
Answer: b. SSRIs

871. The most common form of dementia is:

a. Vascular dementia
b. Alzheimer's disease
c. Lewy body dementia
d. Frontotemporal dementia
Answer: b. Alzheimer's disease

872. The classic triad of symptoms in systemic lupus erythematosus includes:


a. Fever, fatigue, weight loss
b. Joint pain, skin rash, and photosensitivity
c. Hematuria, proteinuria, and edema
d. Abdominal pain, diarrhea, and fever
Answer: b. Joint pain, skin rash, and photosensitivity

873. The preferred initial imaging study for suspected appendicitis in children
is:

a. Ultrasound
b. CT scan
c. MRI
d. X-ray
Answer: a. Ultrasound

874. The primary treatment for seasonal allergic rhinitis is:

a. Antihistamines
b. Corticosteroids
c. Leukotriene receptor antagonists
d. Nasal decongestants
Answer: a. Antihistamines

875. The most common type of lung cancer is:

a. Small cell lung cancer


b. Adenocarcinoma
c. Squamous cell carcinoma
d. Large cell carcinoma
Answer: b. Adenocarcinoma

876. The characteristic sign of Parkinson's disease is:

a. Rigidity
b. Ataxia
c. Hemiparesis
d. Dysmetria
Answer: a. Rigidity

877. The primary cause of hyperparathyroidism is:

a. Adenoma
b. Carcinoma
c. Hyperplasia
d. Vitamin D deficiency
Answer: a. Adenoma

878. The most effective treatment for acute migraine attacks is:

a. NSAIDs
b. Triptans
c. Ergots
d. Antiemetics
Answer: b. Triptans

879. The characteristic sign of acute cholecystitis is:

a. Murphy's sign
b. Courvoisier's sign
c. Kehr's sign
d. Grey Turner's sign
Answer: a. Murphy's sign

880. The most common cause of hyperglycemic hyperosmolar state (HHS) is:

a. Type 1 diabetes
b. Type 2 diabetes
c. Corticosteroid use
d. Acute pancreatitis
Answer: b. Type 2 diabetes

881. The primary treatment for acute otitis media in children is:
a. Antibiotics
b. Observation
c. Antipyretics
d. Surgery
Answer: a. Antibiotics

882. The most common cause of secondary amenorrhea is:

a. Pregnancy
b. Polycystic ovary syndrome (PCOS)
c. Hypothalamic dysfunction
d. Premature ovarian failure
Answer: a. Pregnancy

883. The classic triad of symptoms in diabetes insipidus includes:

a. Polyuria, polydipsia, and hypernatremia


b. Polyuria, hypoglycemia, and weight loss
c. Polyphagia, polydipsia, and weight loss
d. Polydipsia, edema, and hypertension
Answer: a. Polyuria, polydipsia, and hypernatremia

884. The first-line treatment for a confirmed case of streptococcal


pharyngitis is:

a. Amoxicillin
b. Azithromycin
c. Penicillin
d. Cephalexin
Answer: c. Penicillin

885. The classic symptom of chronic obstructive pulmonary disease (COPD)


is:

a. Productive cough
b. Dry cough
c. Hemoptysis
d. Chest pain
Answer: a. Productive cough

886. The most common cause of primary hyperaldosteronism is:

a. Adrenal adenoma
b. Adrenal carcinoma
c. Secondary causes (e.g., renal artery stenosis)
d. Medications
Answer: a. Adrenal adenoma

887. The most common initial presenting symptom of multiple sclerosis is:

a. Numbness or tingling
b. Blurred vision
c. Weakness
d. Gait disturbances
Answer: b. Blurred vision

888. The classic presentation of aortic dissection includes:

a. Sudden onset of severe chest pain


b. Gradual onset of chest pain
c. Pain radiating to the right shoulder
d. Pain with inspiration
Answer: a. Sudden onset of severe chest pain

889. The primary method of prevention for influenza is:

a. Antiviral medications
b. Vaccination
c. Hand hygiene
d. Isolation
Answer: b. Vaccination

890. The most common complication of diabetes mellitus is:


a. Retinopathy
b. Neuropathy
c. Nephropathy
d. Cardiovascular disease
Answer: d. Cardiovascular disease

891. The most common cause of chronic cough in adults is:

a. Asthma
b. Gastroesophageal reflux disease (GERD)
c. Chronic bronchitis
d. Post-nasal drip
Answer: d. Post-nasal drip

892. The preferred imaging study for diagnosing pulmonary embolism is:

a. Chest X-ray
b. CT pulmonary angiography
c. MRI
d. V/Q scan
Answer: b. CT pulmonary angiography

893. The hallmark laboratory finding in hemolytic anemia is:

a. Elevated reticulocyte count


b. Decreased hemoglobin
c. Increased platelet count
d. Elevated white blood cell count
Answer: a. Elevated reticulocyte count

894. The primary risk factor for the development of squamous cell carcinoma
of the skin is:

a. Fair skin
b. Genetic predisposition
c. Chronic sun exposure
d. Smoking
Answer: c. Chronic sun exposure

895. The most effective treatment for allergic rhinitis is:

a. Oral antihistamines
b. Intranasal corticosteroids
c. Decongestants
d. Saline nasal spray
Answer: b. Intranasal corticosteroids

896. The classic presentation of acute kidney injury (AKI) due to prerenal
causes includes:

a. Hematuria
b. Oliguria
c. Elevated blood urea nitrogen (BUN) to creatinine ratio
d. Hyperkalemia
Answer: c. Elevated blood urea nitrogen (BUN) to creatinine ratio

897. The first-line treatment for generalized seizures is

a. Carbamazepine
b. Valproate
c. Phenytoin
d. Levetiracetam
Answer: b. Valproate

898. The most common type of thyroid cancer is:

a. Papillary carcinoma
b. Follicular carcinoma
c. Medullary carcinoma
d. Anaplastic carcinoma
Answer: a. Papillary carcinoma

899. The classic sign of appendicitis is:


a. McBurney's point tenderness
b. Rovsing's sign
c. Psoas sign
d. All of the above
Answer: d. All of the above

900. The most common cardiac arrhythmia encountered in clinical practice is:

a. Atrial fibrillation
b. Ventricular tachycardia
c. Premature ventricular contractions
d. Atrial flutter
Answer: a. Atrial fibrillation

901. The primary cause of secondary hypertension is:

a. Obesity
b. Chronic kidney disease
c. Stress
d. Sedentary lifestyle
Answer: b. Chronic kidney disease

902. The classic triad of symptoms in aortic stenosis includes:

a. Syncope, angina, dyspnea


b. Palpitations, fatigue, chest pain
c. Edema, fatigue, orthopnea
d. Abdominal pain, nausea, jaundice
Answer: a. Syncope, angina, dyspnea

903. The primary pathogen responsible for the majority of


community-acquired pneumonia cases is:

a. Streptococcus pneumoniae
b. Haemophilus influenzae
c. Mycoplasma pneumoniae
d. Legionella pneumophila
Answer: a. Streptococcus pneumoniae

904. The most common congenital heart defect is:

a. Ventricular septal defect (VSD)


b. Atrial septal defect (ASD)
c. Patent ductus arteriosus (PDA)
d. Tetralogy of Fallot
Answer: a. Ventricular septal defect (VSD)

905. The characteristic symptom of hyperparathyroidism is:

a. Constipation
b. Fatigue
c. Bone pain
d. All of the above
Answer: d. All of the above

906. The gold standard for diagnosing diabetes mellitus is:

a. Fasting blood glucose


b. Oral glucose tolerance test (OGTT)
c. Hemoglobin A1c
d. Random blood glucose
Answer: c. Hemoglobin A1c

907. The most common cause of meningitis in adults is:

a. Bacterial infection
b. Viral infection
c. Fungal infection
d. Parasitic infection
Answer: a. Bacterial infection

908. The classic sign of cervical cancer is:


a. Abnormal vaginal bleeding
b. Pelvic pain
c. Vaginal discharge
d. All of the above
Answer: a. Abnormal vaginal bleeding

909. The first-line treatment for obesity is:

a. Behavioral therapy
b. Pharmacotherapy
c. Bariatric surgery
d. Exercise
Answer: a. Behavioral therapy

910. The most common site of ectopic pregnancy is:

a. Ovary
b. Abdomen
c. Fallopian tube
d. Cervix
Answer: c. Fallopian tube

911. The classic symptom of hypothyroidism is:

a. Weight loss
b. Heat intolerance
c. Cold intolerance
d. Increased appetite
Answer: c. Cold intolerance

912. The primary treatment for osteoarthritis is:

a. NSAIDs
b. Corticosteroids
c. Disease-modifying antirheumatic drugs (DMARDs)
d. Opioids
Answer: a. NSAIDs
913. The characteristic finding in rheumatoid arthritis is:

a. Joint erosions
b. Osteophytes
c. Subcutaneous nodules
d. All of the above
Answer: d. All of the above

914. The most common type of stroke is:

a. Ischemic
b. Hemorrhagic
c. Transient ischemic attack (TIA)
d. Cryptogenic
Answer: a. Ischemic

915. The primary treatment for depression in adults is:

a. Cognitive-behavioral therapy
b. SSRIs
c. MAOIs
d. Electroconvulsive therapy
Answer: b. SSRIs

916. The classic symptom of aortic regurgitation is:

a. Dyspnea on exertion
b. Orthopnea
c. Palpitations
d. Wide pulse pressure
Answer: d. Wide pulse pressure

917. The most common cause of hypothyroidism worldwide is:

a. Hashimoto's thyroiditis
b. Iodine deficiency
c. Thyroidectomy
d. Radioactive iodine treatment
Answer: b. Iodine deficiency

918. The first-line treatment for chronic migraine is:

a. Beta-blockers
b. Antidepressants
c. NSAIDs
d. Anticonvulsants
Answer: a. Beta-blockers

919. The most common complication of diabetes is:

a. Neuropathy
b. Retinopathy
c. Nephropathy
d. Cardiovascular disease
Answer: d. Cardiovascular disease

920. The classic symptom of gluten intolerance is:

a. Diarrhea
b. Abdominal pain
c. Weight loss
d. All of the above
Answer: d. All of the above

921. The most common cause of acute pancreatitis is:

a. Alcohol consumption
b. Gallstones
c. Hyperlipidemia
d. Viral infection
Answer: b. Gallstones

922. The primary mechanism of action of beta-blockers is:


a. Inhibition of angiotensin II
b. Blockade of calcium channels
c. Inhibition of beta-adrenergic receptors
d. Decrease in cardiac output
Answer: c. Inhibition of beta-adrenergic receptors

923. The most common symptom of congestive heart failure is:

a. Edema
b. Fatigue
c. Dyspnea
d. Palpitations
Answer: c. Dyspnea

924. The preferred first-line medication for hypertension in a patient with


diabetes is:

a. Beta-blocker
b. Calcium channel blocker
c. ACE inhibitor
d. Diuretic
Answer: c. ACE inhibitor

925. The classic presentation of hyperthyroidism includes:

a. Weight gain and fatigue


b. Heat intolerance and weight loss
c. Cold intolerance and bradycardia
d. Hypotension and lethargy
Answer: b. Heat intolerance and weight loss

926. The primary treatment for anaphylaxis is:

a. Antihistamines
b. Corticosteroids
c. Epinephrine
d. Oxygen therapy
Answer: c. Epinephrine

927. The characteristic feature of ulcerative colitis is:

a. Skip lesions
b. Transmural inflammation
c. Continuous mucosal inflammation
d. Granulomas
Answer: c. Continuous mucosal inflammation

928. The first-line therapy for attention-deficit hyperactivity disorder


(ADHD) in children is:

a. Behavioral therapy
b. Stimulant medications
c. Antidepressants
d. Antipsychotics
Answer: b. Stimulant medications

929. The most common type of lung cancer is:

a. Squamous cell carcinoma


b. Adenocarcinoma
c. Small cell lung cancer
d. Large cell carcinoma
Answer: b. Adenocarcinoma

930. The classic triad of symptoms in pheochromocytoma includes:

a. Headache, palpitations, diaphoresis


b. Hypertension, tachycardia, weight loss
c. Fatigue, anxiety, tremors
d. Nausea, vomiting, abdominal pain
Answer: a. Headache, palpitations, diaphoresis

931. The most common infectious cause of diarrhea in children is:


a. Rotavirus
b. Norovirus
c. Salmonella
d. Shigella
Answer: a. Rotavirus

932. The classic finding in diabetes mellitus is:

a. Elevated fasting glucose


b. Decreased urination
c. Increased weight
d. Normal blood pressure
Answer: a. Elevated fasting glucose

933. The first-line treatment for osteoporosis is:

a. Bisphosphonates
b. Hormone replacement therapy
c. Calcium supplements
d. Vitamin D supplements
Answer: a. Bisphosphonates

934. The most common type of urinary incontinence in elderly women is:

a. Urge incontinence
b. Stress incontinence
c. Overflow incontinence
d. Functional incontinence
Answer: b. Stress incontinence

935. The classic symptom of aortic dissection is:

a. Sudden chest pain radiating to the back


b. Shortness of breath
c. Nausea and vomiting
d. Sweating and palpitations
Answer: a. Sudden chest pain radiating to the back
936. The primary treatment for allergic rhinitis includes:

a. Decongestants
b. Antihistamines
c. Corticosteroids
d. All of the above
Answer: d. All of the above

937. The most sensitive test for diagnosing rheumatoid arthritis is:

a. Anti-citrullinated protein antibody (ACPA)


b. Rheumatoid factor
c. ESR
d. CRP
Answer: a. Anti-citrullinated protein antibody (ACPA)

938. The first sign of iron deficiency anemia is often:

a. Microcytic red blood cells


b. Hypochromic red blood cells
c. Decreased serum ferritin
d. Elevated total iron-binding capacity
Answer: c. Decreased serum ferritin

939. The mainstay of treatment for hyperlipidemia is:

a. Statins
b. Fibrates
c. Niacin
d. Omega-3 fatty acids
Answer: a. Statins

940. The most common cause of a stroke is:

a. Ischemia
b. Hemorrhage
c. Transient ischemic attack
d. Atrial fibrillation
Answer: a. Ischemia

941. The primary treatment for systemic lupus erythematosus (SLE) includes:

a. NSAIDs
b. Corticosteroids
c. Antimalarials
d. All of the above
Answer: d. All of the above

942. The classic sign of chronic liver disease is:

a. Ascites
b. Jaundice
c. Hepatomegaly
d. All of the above
Answer: d. All of the above

943. The most common type of primary headache is:

a. Tension-type headache
b. Migraine
c. Cluster headache
d. Sinus headache
Answer: a. Tension-type headache

944. The most effective method for smoking cessation is:

a. Nicotine replacement therapy


b. Behavioral therapy
c. Prescription medications
d. Cold turkey
Answer: a. Nicotine replacement therapy

945. The typical finding in a patient with Cushing's syndrome is:


a. Weight loss
b. Hyperpigmentation
c. Moon facies
d. Hypotension
Answer: c. Moon facies

946. The most common cause of peptic ulcers is:

a. H. pylori infection
b. Stress
c. NSAID use
d. Excessive alcohol consumption
Answer: a. H. pylori infection

947. The primary symptom of chronic obstructive pulmonary disease (COPD)


is:

a. Chronic cough
b. Wheezing
c. Shortness of breath
d. All of the above
Answer: d. All of the above

948. The standard treatment for tuberculosis is:

a. Rifampin and isoniazid


b. Streptomycin and ethambutol
c. Pyrazinamide and levofloxacin
d. Isoniazid and pyrazinamide
Answer: a. Rifampin and isoniazid

949. The most common cause of hypothyroidism in developed countries is:

a. Hashimoto's thyroiditis
b. Iodine deficiency
c. Surgical removal of the thyroid
d. Radiation therapy
Answer: a. Hashimoto's thyroiditis

950. The primary treatment for panic disorder is:

a. Antidepressants
b. Benzodiazepines
c. Cognitive-behavioral therapy
d. Antipsychotics
Answer: c. Cognitive-behavioral therapy

951. Which of the following conditions is least likely to cause secondary


hypertension?

a. Cushing's syndrome
b. Primary hyperaldosteronism
c. Aortic coarctation
d. Hypothyroidism
Answer: d. Hypothyroidism

952. In a patient with acute respiratory distress syndrome (ARDS), the


hallmark feature on chest X-ray is:

a. Bilateral pleural effusions


b. Ground-glass opacities
c. Consolidation
d. Hyperinflation
Answer: b. Ground-glass opacities

953. A patient presents with sudden onset of severe abdominal pain and
"currant jelly" stools. The most likely diagnosis is:

a. Intussusception
b. Appendicitis
c. Mesenteric ischemia
d. Gastroenteritis
Answer: a. Intussusception
954. Which of the following laboratory findings is most consistent with
chronic kidney disease?

a. Decreased serum creatinine


b. Normal glomerular filtration rate (GFR)
c. Elevated serum phosphorus
d. Hypocalcemia
Answer: c. Elevated serum phosphorus

955. The most likely mechanism of action of ACE inhibitors is:

a. Direct vasodilation
b. Inhibition of angiotensin II production
c. Blockade of beta-adrenergic receptors
d. Inhibition of sodium reabsorption
Answer: b. Inhibition of angiotensin II production

956. In a patient with iron deficiency anemia, the peripheral blood smear
typically shows:

a. Macrocytic red blood cells


b. Hypochromic microcytic red blood cells
c. Schistocytes
d. Acanthocytes
Answer: b. Hypochromic microcytic red blood cells

957. The most significant risk factor for the development of endometrial
cancer is:

a. Early menarche
b. Obesity
c. Oral contraceptive use
d. Nulliparity
Answer: b. Obesity

958. Which of the following is the most effective treatment for a hyperacute
rejection of a transplanted organ?
a. Corticosteroids
b. Antithymocyte globulin
c. Intravenous immunoglobulin
d. Plasmapheresis
Answer: d. Plasmapheresis

959. The presence of anti-double-stranded DNA antibodies is most closely


associated with:

a. Rheumatoid arthritis
b. Systemic lupus erythematosus
c. Sjögren's syndrome
d. Scleroderma
Answer: b. Systemic lupus erythematosus

960. The characteristic finding in a patient with myasthenia gravis is:

a. Muscle atrophy
b. Decreased reflexes
c. Fatigable muscle weakness
d. Paresthesia
Answer: c. Fatigable muscle weakness

961. A patient presents with a "butterfly rash" on the face. This is


characteristic of:

a. Dermatomyositis
b. Systemic lupus erythematosus
c. Psoriasis
d. Eczema
Answer: b. Systemic lupus erythematosus

962. The presence of Charcot's triad (fever, jaundice, and right upper
quadrant pain) suggests:

a. Cholecystitis
b. Acute pancreatitis
c. Ascending cholangitis
d. Hepatitis
Answer: c. Ascending cholangitis

963. The most common cause of hypercalcemia in outpatient settings is:

a. Primary hyperparathyroidism
b. Malignancy
c. Vitamin D intoxication
d. Sarcoidosis
Answer: a. Primary hyperparathyroidism

964. Which of the following drugs is contraindicated in pregnancy due to


teratogenic effects?

a. Acetaminophen
b. Ibuprofen
c. Warfarin
d. Metformin
Answer: c. Warfarin

965. The classic sign of hyperthyroidism is:

a. Bradycardia
b. Weight gain
c. Exophthalmos
d. Cold intolerance
Answer: c. Exophthalmos

966. Which vitamin deficiency is associated with pernicious anemia?

a. Vitamin A
b. Vitamin B1
c. Vitamin B12
d. Vitamin C
Answer: c. Vitamin B12
967. The primary mechanism of action of metformin is:

a. Increased insulin secretion


b. Decreased hepatic glucose production
c. Increased intestinal glucose absorption
d. Decreased peripheral insulin sensitivity
Answer: b. Decreased hepatic glucose production

968. A patient with a history of recurrent kidney stones is found to have


hypercalcemia. The most likely cause is:

a. Primary hyperparathyroidism
b. Familial hypocalciuric hypercalcemia
c. Malignancy
d. Vitamin D overdose
Answer: a. Primary hyperparathyroidism

969. The most effective way to reduce the risk of stroke in patients with atrial
fibrillation is:

a. Antiplatelet therapy
b. Anticoagulation
c. Statin therapy
d. Blood pressure control
Answer: b. Anticoagulation

970. The most common side effect of long-term corticosteroid therapy is:

a. Osteoporosis
b. Hypertension
c. Weight loss
d. Hypoglycemia
Answer: a. Osteoporosis

971. A patient with a history of smoking presents with a persistent cough and
hemoptysis. The most concerning diagnosis is:
a. Chronic bronchitis
b. Lung cancer
c. Tuberculosis
d. Pneumonia
Answer: b. Lung cancer

972. The most likely causative organism in a patient with post-surgical


infection following an appendectomy is:

a. Staphylococcus aureus
b. Escherichia coli
c. Streptococcus pneumoniae
d. Clostridium difficile
Answer: b. Escherichia coli

973. The "M" spike in serum protein electrophoresis is indicative of:

a. Multiple myeloma
b. Waldenström macroglobulinemia
c. Lymphoma
d. Chronic lymphocytic leukemia
Answer: a. Multiple myeloma

974. A common complication of untreated hyperthyroidism is

a. Heart failure
b. Myxedema coma
c. Thyroid storm
d. Osteoporosis
Answer: c. Thyroid storm

975. The most common cause of acute kidney injury in hospitalized patients
is:

a. Prerenal azotemia
b. Intrinsic renal failure
c. Postrenal obstruction
d. Acute tubular necrosis
Answer: d. Acute tubular necrosis

976. In the context of psychiatric disorders, the term "anosognosia" refers to:

a. Inability to recall past events


b. Lack of insight into one's own illness
c. Severe cognitive impairment
d. An inability to express emotions
Answer: b. Lack of insight into one's own illness

977. Which of the following is a classic feature of diabetic ketoacidosis (DKA)?

a. Hyperkalemia
b. Hypoglycemia
c. Metabolic alkalosis
d. Hyponatremia
Answer: a. Hyperkalemia

978. The primary benefit of using statins in patients with hyperlipidemia is:

a. Decreased cholesterol absorption


b. Inhibition of HMG-CoA reductase
c. Increased bile acid excretion
d. Enhanced lipoprotein lipase activity
Answer: b. Inhibition of HMG-CoA reductase

979. A patient presents with signs of Cushing's syndrome. The most


appropriate initial test to confirm the diagnosis is:

a. Serum cortisol levels


b. 24-hour urinary free cortisol
c. Low-dose dexamethasone suppression test
d. Plasma ACTH levels
Answer: b. 24-hour urinary free cortisol

980. In the setting of shock, the term "compensated shock" refers to:
a. Normal blood pressure with tachycardia
b. Hypotension with altered mental status
c. Normal heart rate and blood pressure
d. Hypertension with bradycardia
Answer: a. Normal blood pressure with tachycardia

981. A 55-year-old male presents with new-onset atrial fibrillation. Which of


the following is the most appropriate next step in management?

a. Initiate anticoagulation
b. Perform electrical cardioversion
c. Start beta-blocker therapy
d. Refer for echocardiography
Answer: a. Initiate anticoagulation

982. In a patient with a suspected adrenal crisis, which laboratory finding


would be most characteristic?

a. Hyperkalemia
b. Hypercalcemia
c. Hyponatremia
d. Hyperglycemia
Answer: a. Hyperkalemia

983. Which of the following is a contraindication for the use of nitroglycerin?

a. Acute myocardial infarction


b. Right ventricular infarction
c. Hypertension
d. Asthma
Answer: b. Right ventricular infarction

984. A patient with diabetes presents with worsening neuropathy and foot
ulcers. Which medication is least likely to provide benefit in this scenario?

a. Gabapentin
b. Pregabalin
c. Duloxetine
d. Metformin
Answer: d. Metformin

985. In chronic kidney disease, the term "uremic frost" refers to:

a. Itchy skin
b. Crystalized urea deposits on the skin
c. Electrolyte imbalances
d. Dermatitis due to dialysis
Answer: b. Crystalized urea deposits on the skin

986. Which of the following is the most common cause of


community-acquired pneumonia in adults?

a. Mycoplasma pneumoniae
b. Streptococcus pneumoniae
c. Haemophilus influenzae
d. Legionella pneumophila
Answer: b. Streptococcus pneumoniae

987. In a patient with pulmonary embolism, the most reliable clinical sign is:

a. Hemoptysis
b. Tachypnea
c. Chest pain
d. Hypotension
Answer: b. Tachypnea

988. The term "silent myocardial ischemia" is best defined as:

a. Ischemia without any chest pain


b. Ischemia that occurs only at rest
c. Ischemia that causes vague discomfort
d. Ischemia that leads to heart failure
Answer: a. Ischemia without any chest pain
989. A patient presents with sudden vision loss in one eye. The fundoscopic
examination reveals a pale retina with a characteristic "cherry-red spot."
The most likely diagnosis is:

a. Central retinal artery occlusion


b. Retinal detachment
c. Diabetic retinopathy
d. Acute glaucoma
Answer: a. Central retinal artery occlusion

990. In which of the following conditions would you expect to find


"wire-loop" lesions on renal biopsy?

a. Minimal change disease


b. Diabetic nephropathy
c. Systemic lupus erythematosus
d. Hypertensive nephrosclerosis
Answer: c. Systemic lupus erythematosus

991. The classic triad of symptoms for a pheochromocytoma includes all


EXCEPT:

a. Hypertension
b. Tachycardia
c. Hypoglycemia
d. Headaches
Answer: c. Hypoglycemia

992. In patients with hepatitis C, the most effective treatment regimen


currently available is:

a. Interferon monotherapy
b. Ribavirin and interferon
c. Direct-acting antiviral agents
d. Supportive care only
Answer: c. Direct-acting antiviral agents
993. Which type of hypersensitivity reaction is involved in anaphylaxis?

a. Type I
b. Type II
c. Type III
d. Type IV
Answer: a. Type I

994. A patient with newly diagnosed type 2 diabetes is started on metformin.


What is the most important adverse effect to monitor for?

a. Lactic acidosis
b. Gastrointestinal upset
c. Weight gain
d. Hypoglycemia
Answer: a. Lactic acidosis

995. Which of the following cancers is most associated with the


paraneoplastic syndrome of hypercalcemia?

a. Breast cancer
b. Lung cancer
c. Squamous cell carcinoma
d. Renal cell carcinoma
Answer: c. Squamous cell carcinoma

996. In patients with rheumatoid arthritis, which joint is typically spared in


the early stages of the disease?

a. Metacarpophalangeal joint
b. Wrist joint
c. Distal interphalangeal joint
d. Proximal interphalangeal joint
Answer: c. Distal interphalangeal joint

997. The most common inherited bleeding disorder is:


a. Hemophilia A
b. von Willebrand disease
c. Hemophilia B
d. Thrombocytopenia
Answer: b. von Willebrand disease

998. A 70-year-old man presents with sudden onset of confusion and focal
neurological deficits. A CT scan shows a hyperdense middle cerebral artery
sign. This indicates:

a. Ischemic stroke
b. Hemorrhagic stroke
c. Transient ischemic attack
d. Brain tumor
Answer: b. Hemorrhagic stroke

999. The primary mechanism of action of furosemide is:

a. Inhibition of sodium-potassium-ATPase
b. Inhibition of sodium reabsorption in the distal convoluted tubule
c. Inhibition of sodium reabsorption in the loop of Henle
d. Increased renal blood flow
Answer: c. Inhibition of sodium reabsorption in the loop of Henle

1000. A patient presents with a rash that appears after exposure to sunlight,
along with joint pain and fatigue. This presentation is suggestive of:

a. Psoriasis
b. Eczema
c. Systemic lupus erythematosus
d. Dermatomyositis
Answer: c. Systemic lupus erythematosus

You might also like